möclöc - geosiro.com · sßdöngph÷ìngph¡p˜⁄isŁtrong chøngminhh…nhhåc nguy„n tăng...

60
Mc lc 1 Sß dng phương pháp đ/i sL trong chøng minh hình hc 3 2 BŒ đ• v• sL mũ đúng 19 3 Đa thøc b§t kh£ quy 27 4 Các bài toán tŒ hæp trên dãy sL 41 5 Đ• thi tham kh£o hưng ti VMO 2019 50 1

Upload: others

Post on 23-Jan-2020

5 views

Category:

Documents


0 download

TRANSCRIPT

Page 1: Möclöc - geosiro.com · Sßdöngph÷ìngph¡p˜⁄isŁtrong chøngminhh…nhhåc Nguy„n Tăng Vũ (Giáo viên PTNK TP Hç Chí Minh) Bài toán hình håc phflng thưíng xoay

Mục lục

1 Sử dụng phương pháp đại số trong chứng minh hình học 3

2 Bổ đề về số mũ đúng 19

3 Đa thức bất khả quy 27

4 Các bài toán tổ hợp trên dãy số 41

5 Đề thi tham khảo hướng tới VMO 2019 50

1

Page 2: Möclöc - geosiro.com · Sßdöngph÷ìngph¡p˜⁄isŁtrong chøngminhh…nhhåc Nguy„n Tăng Vũ (Giáo viên PTNK TP Hç Chí Minh) Bài toán hình håc phflng thưíng xoay

2

NGUYỄN TĂNG VŨ - NGUYỄN NGỌC DUY

VƯƠNG TRUNG DŨNG - LÊ PHÚC LỮ

TẬP SAN TOÁN HỌC

STAR EDUCATIONSố thứ 01 - 2019

Phong trào chuyên Toán các năm gần đây diễn ra ngày một sôi nổi trên khắp cả nước;cùng với đó là nhiều chuyên tạp chí, kỷ yếu, chuyên đề liên quan đến Toán sơ cấpđược phát hành rộng rãi.

Để góp phần vào phong trào ấy, STAR EDUCATION ra mắt cuốn tập san cho riêngmình dành tặng các bạn đồng nghiệp, học sinh đam mê với môn Toán, giúp các em cóthêm nhiều kinh nghiệm trong các kì thi HSG cũng như các kì thi toán khác. Dự địnhcủa chúng tôi là một năm sẽ có hai quyển là:

• Quyển 1: ra mắt vào đầu năm, hướng tới kỳ thi HSG quốc gia.

• Quyển 2: ra mắt vào giữa năm, hướng tới kỳ thi tuyển sinh THPT, Đại học.

Trong tài liệu đợt này, chúng tôi giới thiệu các bài giảng chọn lọc cùng 1 đề thithử tham khảo. Lần đầu ra mắt, tác giả các bài viết chính là giáo viên của STAREDUCATION, rất mong sự đón nhận của bạn đọc các nơi và mong có thêm nhiều bàiviết gửi về STAR EDUCATION để thêm phần phong phú.

Ban biên tập dù đã cố gắng rất nhiều nhưng không tránh khỏi sai sót, xin nhận đượcgóp ý từ đồng nghiệp cùng các em học sinh. Mong rằng tài liệu này sẽ có ích chocác thí sinh đang chuẩn bị cho kỳ thi sắp tới. Mọi thắc mắc, đóng góp xin liên hệ[email protected].

Tập san Toán học STAR EDUCATION

Page 3: Möclöc - geosiro.com · Sßdöngph÷ìngph¡p˜⁄isŁtrong chøngminhh…nhhåc Nguy„n Tăng Vũ (Giáo viên PTNK TP Hç Chí Minh) Bài toán hình håc phflng thưíng xoay

Sử dụng phương pháp đại số trongchứng minh hình học

Nguyễn Tăng Vũ(Giáo viên PTNK TP Hồ Chí Minh)

Bài toán hình học phẳng thường xoay quanh việc chứng minh 3 điểm thẳng hàng, các điểm đồngviên, sự tiếp xúc, các đẳng thức, bất đẳng thức hình học...Có nhiều phương pháp tiếp cận nhưphương tích trục đẳng phương, hàng điểm điều hòa, cực đối cực, phép biến hình, tính toán. Trongđó phương pháp tính toán cũng có khá nhiều cách như tọa độ Oxy, số phức, tọa độ cực,...Việctính toán trong hình học, thoạt nhìn có vẻ không đẹp, lời giải cồng kềnh không mang nhiều ýnghĩa hình học thuần túy. Người làm toán hay các học sinh giỏi thường ngại sử dụng phươngpháp này, tuy nhiên trong lúc cần một công cụ để giải nhanh bài toán thì phương pháp này lạihiệu quả. Vì muốn đẹp về mặc hình học thì cần phải vẽ thêm nhiều yếu tố phụ ẩn chứa sau môhình, với việc tính toán ta giảm đi việc phát hiện yếu tố phụ và chỉ sử dụng các yếu tố có sẵn.Hơn nữa việc tính toán phức tạp với định hướng rõ ràng sẽ rất có lợi sau này trong việc nghiêncứu các vấn đề khác. Trong bài viết này tôi đưa ra một phương pháp tính gần gũi với hình họcthuần túy đó là việc sử dụng các định lý pitago, định lý sin, định lý cosin, ptoleme trong việc tínhtoán các độ dài, hay tỉ số của các đoạn thẳng, bên cạnh đó kết hợp các định lý Ceva, Menelaus đểchứng minh các bài toán thẳng hàng, đồng viên.

1. Lý thuyết và ví dụ

Ngoài các định lý Ceva, Menelaus dạng đại số, lượng giác ta còn sử dụng các định lýsau:

Định lý 1. (Định lý Sin) Cho tam giác ABC , đặt BC = a, AC = b, AB = c, R là bánkính đường tròn ngoại tiếp tam giác. Khi đó

asin A

=b

sin B=

csin C

= 2R

Chú ý Định lý sin rất hiệu quả trong việc chuyển một tỉ lệ về độ dài sang một tỉ lệ vềlượng giác, từ đó có thể biến đổi lượng giác.

Định lý 2. (Định lý Cosin) Trong tam giác ta có a2 = b2 + c2 − 2bc cos A.

3

Page 4: Möclöc - geosiro.com · Sßdöngph÷ìngph¡p˜⁄isŁtrong chøngminhh…nhhåc Nguy„n Tăng Vũ (Giáo viên PTNK TP Hç Chí Minh) Bài toán hình håc phflng thưíng xoay

4 NGUYỄN TĂNG VŨ

Định lý 3. (Ptolemy) Cho 4 điểm A, B, C , D. Khi đó

AB.C D+ AD.BC ≥ AC .BD

Đẳng thức xảy ra khi và chỉ khi ABC D là tứ giác nội tiếp.

Định lý 4. Cho tam giác ABC . Khi đó:

1. SABC =12

AB.AC sin BAC .

2. M là điểm trên cạnh BC , khi đóBM

CAM=

AB. sin MABAC . sin MAC

. Nếu M là trung điểm

BC thìABAC=

sin MACsin MAB

.

Chú ý Tính chất này là một trong những tính chất được sử dụng khi cần chứng minhmột đường thẳng đi qua trung điểm của một đoạn thẳng nào đó.

Tính chất 1. Cho tam giác ABC cân tại A, M là điểm thuộc cạnh BC . Khi đó:

MBMC=

sin MABsin MAC

Tính chất 2. Cho 0o ≤ x , y ≤ α < 90o thỏasin x

sin(α− x)=

sin ysin(α− y)

thì x = y .

Có một số bổ đề định lý các bạn đã biết được cách chứng minh bằng những phươngpháp hình học thuần túy như sử dụng tam giác đồng dạng, phép biến hình như địnhlý Feuerbach, định lý Protassov cho lời giải rất ngắn gọn, nhưng để có được lời giảiđó là điều không dễ dàng nghĩ ra. Trong các vị dụ đầu tiên này, chúng tôi đưa ra cácchứng minh chủ yếu dựa vào tính toán đại số.

Ví dụ 1. Cho tam giác ABC , đường tròn tâm I nội tiếp tam giác ABC tiếp xúc vớiBC , AC , AB tại D, E, F . DI cắt EF tại K . Chứng minh AK qua trung điểm của BC .

Bổ đề này có thể chứng minh dựa vào tứ giác nội tiếp, bằng cách vẽ đường thẳng quaK song song với BC cắt AB, AC tại P,Q và chứng minh tam giác I PQ cân. Đó là cáchmà bất kì học sinh giỏi nào cũng làm được. Tuy nhiên để giải được ta phải biết vẽđường phụ, và đó là trở ngại lớn nhất đối với học sinh. Ta hãy giải bài toán này theohướng tiếp cận đại số.

Chứng minh. Gọi M là trung điểm BC , ta sẽ chứng minh tia AK trùng tia AM . Từtính chất 2, ta thấy rằng để chứng minh 2 tia này trùng nhau, ta chỉ cần chứng minhsin∠BAKsin∠CAK

=sin∠BAMsin∠CAM

(*)

Tập san Toán học STAR EDUCATION

Page 5: Möclöc - geosiro.com · Sßdöngph÷ìngph¡p˜⁄isŁtrong chøngminhh…nhhåc Nguy„n Tăng Vũ (Giáo viên PTNK TP Hç Chí Minh) Bài toán hình håc phflng thưíng xoay

NGUYỄN TĂNG VŨ

Áp dụng tính chất 1, ta có:ABAC=

sin Csin B

=sin∠KI Esin∠KI F

=KEKF=

sin∠KAEsin∠KAF

. Mặt khác, ta

cũng cóABAC=

sin∠MABsin∠MAC

. Từ đó ta có:

sin∠KAEsin∠KAF

=sin∠MAEsin∠MAF

.

Tiếp theo là định lý Casey - Một mở rộng của định lý Ptoleme, đây là một định lývới phát biểu và hình thức khá phức tạp, tuy vậy để giải nó ta chỉ cần các công cụtính toán đơn giản là định lý cosin và định lý Pitago, tất nhiên phải cần đến định lýPtoleme.

Ví dụ 2. (Định lý Casey - Mở rộng của định lý Pltolemy) Cho tứ giác ABCD nội tiếp(O, R). Đặt các đường tròn α,β ,γ,δ là các đường tròn tiếp xúc với (O) tại các đỉnhA, B, C, D. Đăt tαβ là độ dài đoạn tiếp tuyến chung của hai đường tròn α,β . Trongđó tαβ là độ dài tiếp tuyến chung ngoài nếu hai đường tròn α,β cùng tiếp xúc tronghoặc cùng tiếp xúc ngoài với (O), và là độ dài đoạn tiếp xúc trong với trường hợp cònlại. Các đoạn tαγ, tβγ, ... được xác định tương tự. Khi đó ta có

tαβ .tγδ + tβγ.tαδ = tαγ.tβδ.

Lời giải. 1. Gọi (A′; r1), (B′, r2), (C ′, r3), (D′, r4) lần lượt là các đường tròn tiếp xúcvới (O) tại các tiếp điểm A, B, C , D.Đặt a = AB, b = BC , c = C D, d = DA, x = AC , y = BD.

2. Ta có t2αβ= A′B′2 − (r2 − r1)2. Theo định lý cosin ta có A′B′ = OA′2 + OB′2 −

2OA′.OB′. cos∠A′OB′ = (r + r1)2 + (r + r2)2 − 2(r + r1)(r + r2) cos∠A′OB′.

Khi đó t2αβ= 4(r + r1)(r + r2) sin

2 AOB2

, suy ra tαβ = arp

(r + r1)(r + r2).

Tập san Toán học STAR EDUCATION

Page 6: Möclöc - geosiro.com · Sßdöngph÷ìngph¡p˜⁄isŁtrong chøngminhh…nhhåc Nguy„n Tăng Vũ (Giáo viên PTNK TP Hç Chí Minh) Bài toán hình håc phflng thưíng xoay

6 NGUYỄN TĂNG VŨ

3. Tương tự ta có tβγ = brp

(r + r2)(r + r3), tγδ = crp

(r + r3)(r + r4), tαδ =drp

(r + r1)(r + r4).Và tαγ = x r

p

(r + r1)(r + r3), tβδ = y rp

(r + r2)(r + r4).

4. Từ đó ta có (∗)⇔ ac + bd = x y (Đúng do định lý Ptoleme).

Ví dụ 3. (Đường thẳng Newton) Cho các tứ giác ABC D ngoại tiếp đường tròn (I).Gọi E, F, G, H là tiếp điểm của (I) với các cạnh AB, BC , C D, DA; M , N là trung điểmcủa AC và BD.

1. Chứng minh AC , BD, EG, FH đồng quy.

2. Chứng minh I , M , N thẳng hàng vàI MIN=

BE + DHAE + CH

.

Lời giải. Đặt AE = AH = a, BE = BF = b, C F = CG = c, DG = DH = d.

Tập san Toán học STAR EDUCATION

Page 7: Möclöc - geosiro.com · Sßdöngph÷ìngph¡p˜⁄isŁtrong chøngminhh…nhhåc Nguy„n Tăng Vũ (Giáo viên PTNK TP Hç Chí Minh) Bài toán hình håc phflng thưíng xoay

NGUYỄN TĂNG VŨ

(a) Gọi K là giao điểm của EG và AC . Ta cóAKAE=

sin∠AEKsin AKE

vàCKCG=

sin∠CGKsin∠CKG

. Mà

sin∠AKE = sin∠CKG, sin∠AEK = sin CGK .

Do đóAKCK=

AECG=

ac

. Gọi K ′ là giao điểm của HF và AC ta cũng chứng minh được

AK ′

CK ′=

ac

. Do đó K ≡ K ′ hay EG, HF, AC đồng quy. Tương tự ta cũng có BD, EG, HF

đồng quy.

(b) Ta có AB.−→I E = b.

−→IA+ a.

−→IB, BC .

−→I F = b.

−→IC + c.

−→IB, C D.

−→IG = c.

−→I D + d.

−→IC , AD.

−→IH =

d.−→IA+ a.

−→I D. Theo định lý con nhím ta có

AB.−→I E + BC .

−→I F + C D.

−→IG + AD.

−→IH =

−→0 ,

suy ra (a+c).(−→IB+−→I D)+(b+d).(

−→IA+−→IC) =

−→0 . Mà

−→IA+−→IC = 2

−→I M ,−→IB+−→I D = 2

−→IN . Do

đó (a+ c).−→IN +(b+d).

−→I M =

−→0 , từ đó suy ra I , M , N thẳng hàng và

I MIN=

b+ da+ c

.

Ví dụ 4. (Định lý Feuerbach) Trong một tam giác thì đường tròn Euler tiếp xúc trongvới đường tròn nội tiếp và tiếp xúc ngoại với các đường tròn bàng tiếp.

Lời giải. Gọi E, O, I lần lượt là tâm đường tròn Euler, ngoại tiếp, nội tiếp tam giácABC . SQ là đường kính vuông góc BC tại M . P là hình chiếu của A trên PS; gọiK , N , F là hình chiếu của A, I , E trên BC . Ta có E là trung điểm OH, nên F là trungđiểm MK .Ta thấy trong tam giác IHO thì I E là trung tuyến, một hướng suy nghĩ tự nhiên làtính I E theo IH, IO và OH, các đoạn thẳng này đều có thể biểu diễn theo p, R, r, tuynhiên cách tiếp cận này bắt buộc tính rất nhiều đẳng thức liên quan đến các độ dài.Trong bài viết này, trình bày một các khác, do áp dụng với một số tính chất hình họcphẳng nên cách tính sẽ nhẹ nhàng hơn, do chỉ dùng pitago cho tam giác I EL, với L làgiao điểm của IN và HO, tất nhiên các đoạn thẳng I L, LE cũng cần liên hệ với R, r.

Tập san Toán học STAR EDUCATION

Page 8: Möclöc - geosiro.com · Sßdöngph÷ìngph¡p˜⁄isŁtrong chøngminhh…nhhåc Nguy„n Tăng Vũ (Giáo viên PTNK TP Hç Chí Minh) Bài toán hình håc phflng thưíng xoay

8 NGUYỄN TĂNG VŨ

1. Chứng minh OP = 2EF . Ta có AH = 2OM và OM + KH = 2EF và AK = PM .Khi đó OP = PM −OM = AK −OM = AH +HK −OM = OM +HK = 2EF .

2. Chứng minh IN .PQ = MN .NK .Ta có ∠KAS = ∠N I D = ∠PAQ = ∠ASQ, đặt là α. Mà IN = I D cosα, PQ =AP tanα = AS sinα tanα, suy ra IN .PQ = I D.AS sin2α. Mặt khác, MN .NK =IS sinα.AI sinα= IS.AI sin2α. Mà SD.SA= SM .SQ = SB2 = IS2 nên suy ra

I D.AS = (IS − SD)(IS + IA) = IS2 − SD(IS + IA) + IS.IA= IS.IA

Từ đó ta được IN .PQ = MN .NK .

3. Chứng minh I E =R2− r. Ta có

I E2 = (IN − EF)2 + (MN −M F)2 = (IN =OP2)2 + (MN −

MK2)2

=OP2

4+ IN 2 − IN .OP +

MK2

4+MN 2 −MN .MK

=OA2

4+ IN 2 − IN .OP −MN .NK =

R2

4+ r2 − IN(OP + PQ)

=R2

4+ r2 − R.r = (

R2− r)2.

Do đó I E =R2− r nên ta có (I) và (E) tiếp xúc trong.

Ví dụ 5. (Định lý Protasov) Cho tam giác ABC , một đường tròn w qua A, C cắt AB, BCtại D, E. Một đường tròn w′ tiếp xúc với AD, AE và cung DE tại M . Chứng minh rằngphân giác góc ∠AMC đi qua tâm đường tròn nội tiếp của tam giác ABC .

Lời giải. Gọi N là điểm chính giữa cung AC , N I cắt cung DE tại M ′. Ta chứng minhM ′ ≡ M .Vẽ tiếp tuyến của (O) tại M ′ cắt AB, BC tại P,Q. Ta cần chứng minh M ′ là tiếp điểmcủa w′ và w hay M ′ là tiếp điểm của đường tròn nội tiếp tam giác BPQ và PQ. Từ đó

cần chứng minhM ′PM ′Q

=cot∠BPQcot BQP

.

Bài toán đưa về việc tính toán tỉ số lượng giác của các góc.

• Đặt a = ∠IAM ′, x = ∠MAB; b = ∠IC M ′, y = ∠M ′CB.Khi đó ∠BPQ = 2b+ y + x ,∠BQP = ∠2a+ x + y .

• Ta cóAM ′

sin∠APM ′=

PM ′

sin M ′AP, suy ra PM ′ =

AM ′. sin xsin(2b+ x + y)

.

Tương tự QM ′ =C M ′ sin y

sin(2a+ x + y).

Suy raPM ′

QM ′=

AM ′

C M ′.sin x sin(2a+ x + y)sin y sin(2b+ x + y)

=sin x sin(2b+ y) sin(2a+ x + y)sin y sin(2a+ x) sin(2b+ x + y)

.

Tập san Toán học STAR EDUCATION

Page 9: Möclöc - geosiro.com · Sßdöngph÷ìngph¡p˜⁄isŁtrong chøngminhh…nhhåc Nguy„n Tăng Vũ (Giáo viên PTNK TP Hç Chí Minh) Bài toán hình håc phflng thưíng xoay

NGUYỄN TĂNG VŨ

• Ta cần chứng minhsin x sin(2b+ y) sin(2a+ x + y)sin y sin(2a+ x) sin(2b+ x + y)

=cot(

2b+ x + y2

)

cot(2a+ x + y

2). (1)

• Sau khi biến đổi lượng giác ta có (1) tương đương:sin x sin(2b+ y) cos(2a+x+ y)−sin y sin(2a+x) cos(2b+x+ y) = sin y sin(2a+x)− sin x sin(2b+ y).

• Áp dụng biến tích thành tổng liên tục ta có:RH = sin x sin(2b + y) cos(2a + x + y) − sin y sin(2a + x) cos(2b + x + y) =2(sin b sin(a+ x) cos b cos(a+ x)− sin a sin(b+ y) cos a cos(b+ y). (2)

• Mặt khác áp dụng Ceva sin cho tam giác AM ′C với AI , C I , M ′I và vì ∠AM ′I =∠C M ′I nên:sin a sin(b+ y) = sin b sin(a+ x). (3).

• Từ (2), (3) ta có:RHS = (cos b sin(b+ y)(sin(2a+x)−sin x)−sin b cos(b+ y)(sin(2a+x)+sin x) =sin(2a+x)(sin(b+y) cos b−sin b cos(b+y))−sin x(sin(b+y)−sin b cos(b+y)) =sin(2a+ x) sin y − sin x sin(2b+ y) = LHS.

Ví dụ 6. Cho tam giác ABC có BB′, CC ′ là các đường phân giác trong của góc B, C .Chứng minh rằng:

B′C ′ ≥2bc

(a+ b)(a+ c)

(a+ b+ c) sinA2−

a2

Lời giải. • Áp dụng bất đẳng thức Ptoleme cho tứ giác BCB′C ′ ta có B′C ′ ≥BB′.CC ′ − BC ′.B′C

BC(1)

Tập san Toán học STAR EDUCATION

Page 10: Möclöc - geosiro.com · Sßdöngph÷ìngph¡p˜⁄isŁtrong chøngminhh…nhhåc Nguy„n Tăng Vũ (Giáo viên PTNK TP Hç Chí Minh) Bài toán hình håc phflng thưíng xoay

10 NGUYỄN TĂNG VŨ

• Ta có BB′ =2ac cos

∠B2

a+ cvà cos

∠2

cos∠C2=

a+ b+ c2a

sin∠A2

.

• Khi đó BB′.CC ′ =4a2 bc

(a+ c)(a+ b)cos∠B2

cos∠C2=

2abc(a+ b+ c)(a+ b)(a+ c)

sin∠A2

. (2)

• Hơn nữa BC ′ =ac

a+ b, CB′ =

aba+ b

, suy ra BC ′.CB′ =a2 bc

(a+ b)(a+ c)(3)

• Từ (1), (2) và (3) ta có điều cần chứng minh.

Ví dụ 7. Cho tam giác ABC nhọn có trực tâm H. Gọi M là trung điểm BC , đườngtròn tâm M bán kính MH cắt BC tại A1, A2; các điểm B1, B2, C1, C2 được xác địnhtương tự. Chứng minh rằng 6 điểm A1, A2, B1, B2, C1, C2 cùng thuộc một đường tròn.

Lời giải. Ta dễ nhận ra rằng các điểm này cách đều tâm đường tròn ngoại tiếp tamgiác ABC , vậy ta chỉ cần tính OA1 sao cho không phụ thuộc vào vị trí của A1, hay kếtquả là một biểu thức đối xứng ta sẽ có điều cần chứng minh.

Ta có OA21 = OM2 +MA2

1 = OM2 +MH2 và

MH2 =12(HB2+HC2)−

14

BC2 = 2ON 2+2OP2−14

a2 = R2(2cos2 B+2 cos2 C−sin2 A).

Khi đó

OA21 = R2(2cos2 B + 2 cos2 C + cos2 A− sin2 A) = R2(2cos2 B + 2cos2 C + 2 cos2 A− 1).

Tương tự cho các độ dài khác, từ đó ta có 6 điểm thuộc đường tròn tâm O.

Tập san Toán học STAR EDUCATION

Page 11: Möclöc - geosiro.com · Sßdöngph÷ìngph¡p˜⁄isŁtrong chøngminhh…nhhåc Nguy„n Tăng Vũ (Giáo viên PTNK TP Hç Chí Minh) Bài toán hình håc phflng thưíng xoay

NGUYỄN TĂNG VŨ

Ví dụ 8. Cho tam giác ABC nội tiếp đường tròn (O). Tiếp tuyến tại B, C cắt nhau tạiL. Gọi X là điểm đối xứng của A qua BC , tiếp tuyến tuyến tại A cắt LX tại K . Chứngminh K thuộc đường thẳng Euler của tam giác ABC .

Lời giải. • Gọi giao điểm của OK với AX là J , ta sẽ chứng minh J là trực tâm của

∆ABC . Gọi giao điểm của OL với AK là I , theo định lý Thales ta cóJAOI=

KJKO=

JXOL⇔

JAJX=

OIOL

.

• Gọi H là trực tâm của tam giác ABC và P là giao của AH và (O), do tính đối

xứng thì AP = HX . Ta cần chứng minhHAHX=

JAJX

, tức làAHAP=

OIOL

(1).

• Từ đây chú ý thêm ∠OIA= 90o −∠OAH = ∠AC P = α, hướng giải quyết của ta

đã sáng sủa hơn, ta có : OI =OA

sinα; OL =

OCcos∠BAC

⇒OIOL=

cos∠BACsinα

• Ta có AH = 2R cos∠BAC; AP = 2R. sinα, suy raAHAP=

cos∠BACsinα

=OIOL

. Suy ra

HAHX=

JAJX

; nghĩa là H trùng J , suy ra K thuộc đường thẳng Euler của ∆ABC .

Sau đây là bài toán chọn đội tuyển Việt Nam năm 2009, bài này có nhiều cách tiếpcận, trong bài viết này đưa ra 2 cách. Cách thứ nhất do bạn Hồ Quốc Đăng Hưng giải,dựa vào yếu tố phụ và một chút biến đổi tỉ số cạnh và tỉ số lượng giác. Cách thứ 2thực dụng hơn, không cần yếu tố phụ nên chỉ sử dụng thuần túy biến đổi lượng giác.Các bạn cùng xem nhé.

Ví dụ 9. (VN TST 2009) Cho đường tròn (O) đường kính AB, M là một điểm nằmtrong (O). Phân giác trong ∠AMB cắt (O) tại N (N khác phía M đối với AB). Phângiác ngoài ∠AMB cắt NA, NB lần lượt tại P,Q. AM cắt đường tròn đường kính NQtại điểm thứ 2R. BM cắt đường tròn đường kính N P tại điểm thứ 2S. Chứng minhtrung tuyến kẻ từ N của tam giác NSR luôn đi qua một điểm cố định.

Tập san Toán học STAR EDUCATION

Page 12: Möclöc - geosiro.com · Sßdöngph÷ìngph¡p˜⁄isŁtrong chøngminhh…nhhåc Nguy„n Tăng Vũ (Giáo viên PTNK TP Hç Chí Minh) Bài toán hình håc phflng thưíng xoay

12 NGUYỄN TĂNG VŨ

Lời giải. Cách 1: Hướng đi và bình luận của Hồ Quốc Đăng Hưng

Qua hình vẽ ta có thể xác định được ngay điểm cố định chính là O. Thay vì chứngminh trực tiếp trung tuyến kẻ từ N của ∆NSR đi qua O, ta có thể chứng minh NO điqua trung điểm của SR.

Nhưng cách xác định điểm S và R gây khá nhiều khó khăn và ta gần như không thểtìm được mối liên hệ nào giữa trung điểm SR với các yếu tố còn lại. Để giải quyết khó

khăn này, ta nên chuyển hướng sang chứng minhsin∠RNOsin∠SNO

=NSNR

(*), một hệ thức

không liên quan gì đến trung điểm SR. Vấn đề bây giờ là các góc ∠RNO,∠SNO. Ta sẽtìm một số tính chất từ giả thiết.

Ta có ∠BMQ = ∠RMQ, mà ∠RMQ = ∠RNQ do R,Q, N , M cùng thuộc đường trònđường kính NQ, nên ∠BMQ = ∠RNQ. Hoàn toàn tương tự, ta cũng có ∠AM P =∠PNS, mà ∠AM P = ∠RMQ, cho nên ta suy ra được

∠RMQ = ∠BMQ = ∠RNQ = ∠PNS = ∠PMA= ∠PMS = α.

Mặt khác, ∠RNO = ∠BNO−∠BNR= ∠OBN −α (1)Đến đây để ý rằng: ∠MBN = ∠BMQ +∠BQM ⇔ ∠OBN +∠MBA= α+∠BQM ⇒∠OBN −α= ∠BQM −∠MBA (2).Từ (1), (2), suy ra ∠RNO = ∠BQM −∠MBA (3).

Đến đây, bài toán vẫn chưa tiến triển nhiều, ta phải tìm cách tính ∠RNO tường minhhơn nữa bằng cách vẽ thêm yếu tố phụ.

Từ hệ thức (3) ta thấy rằng nếu trên đoạn AB ta lấy điểm L sao cho ∠M LA= ∠MQBthì ta sẽ có ∠BM L = ∠M LA−∠MBA= ∠BQM −∠MBA= ∠RNO (*). Hơn nữa, vớicách dựng điểm L như thế thì tứ giác MQBL nội tiếp. Đến đây để ý rằng ∠M LA =∠MQB = ∠MNA, ta có thêm tính chất M , L, N , A đồng viên. Như vậy dù chưa phântích cụ thể nhưng ta có thể tự tin với cách lấy thêm điểm L này, vì ta vừa có được liênhệ với ∠RNO, vừa có thêm các tính chất đồng viên tương ứng.

Vậy ta lấy điểm L trên đoạn AB xác định như trên. Đến đây, một hướng đi rất tự nhiênlà ta sẽ dựng thêm 1 điểm K tương tự để liên hệ với góc ∠SNO, tuy nhiên nếu tínhtoán kỹ hơn ta sẽ thấy điều này không cần thiết.

Tập san Toán học STAR EDUCATION

Page 13: Möclöc - geosiro.com · Sßdöngph÷ìngph¡p˜⁄isŁtrong chøngminhh…nhhåc Nguy„n Tăng Vũ (Giáo viên PTNK TP Hç Chí Minh) Bài toán hình håc phflng thưíng xoay

NGUYỄN TĂNG VŨ

Ta có ∠SNR = ∠ANB − ∠PNS − ∠RNQ = 90o − 2α; ∠LNB = ∠ANB − ∠AN L =90o − (180o − ∠LMA) = 90o − (∠PMA+ ∠BMQ + ∠BM L) = 90o − 2α − ∠BM L =∠SNR−∠BM L = ∠SNR−∠RNO = ∠SNO (theo (**) ta có ∠BM L = ∠SNO).

Vậy ta có ∠LNB = ∠SNO. Đến đây do tứ giác MQBL nội tiếp ta có ∠LQB = ∠BM L =

∠RNO. Lúc này, ta hoàn toàn có thể dựa vào tam giác LNQ để tínhsin∠RNOsin∠SNO

=

sin∠LQBsin∠LNB

=LNLQ

. Theo (*), ta chỉ cần chứng minhLNLQ=

NSNR

nữa là xong.

Mà do 2 tam giác vuông PSN và QRN đồng dạng (cùng có một góc nhọn α) nênNSNR=

N PNQ

. Điểm P không có nhiều liên hệ với L, ta sẽ thayN PNQ

bởiMNMQ

. Vậy ta đưa

hệ thức (*) ban đầu về chứng minhLNLQ=

MNMQ

. (***)

Ta hoàn toàn có thể làm cho hệ thức này đơn giản hơn nữa bằng cách lợi dụng 2 tứgiác nội tiếp MQBL và AM LN . Áp dụng cho đường tròn (AM LN) thì

LNMN

=sin∠BANsin∠MAN

.

Lại áp dụng tương tự cho đường tròn (M LBQ) ta có

LQMQ=

sin∠LBQsin∠MBQ

=sin∠ABNsin∠MBN

Từ các điều trên, ta suy ra bài toán sẽ được giải quyết nếu ta chứng minh được

sin∠BANsin∠MAN

=sin∠ABNsin∠MBN

⇔NBNA=

sin∠MANsin∠MBN

,

một hệ thức chỉ phụ thuộc vào các điểm M , N , A, B ban đầu. Những thao tác còn lạitương đối đơn giản, ta có:

NAsin∠AMN

=MN

sin∠MAN;

NBsin∠BMN

=MN

sin∠MBN

Từ đó, kết hợp với ∠AMN = ∠BMN , ta suy ra (***) và (*) đúng, theo kết quả 2, 3, tacó trung tuyến từ N của ∆NSR đi qua O cố định.

Lời giải. Cách 2. (Lượng giác hóa) Cũng tương tự cách trên, ta cần chứng minhsin ONSsin RNO

=NRNS

, tuy nhiên cách giải này chuyển hết sang các góc liên quan để tính.

Đặt x = ∠NAB, y = ∠BNA, a = ∠MAB, b = ∠MBA,α= ∠SN P = ∠RNQ.Khi đó ta có

x + y = 90o,∠PMA= α,∠M PA= a+ x −α,∠MQB = b+ y −α, a+ b = 2α.

Ta cóSMAN

SMBN=

MAMB=

MA. sin(a+ x).ANMB. sin(b+ y).BN

, suy rasin(a+ x)sin(b+ y)

=BNAN=

sin xsin y

(1).

Ta cần chứng minhsin ONSsin RNO

=NRNS⇔

sin(x −α)sin(y −α)

=sin(x + a−α)sin(y + b−α)

. (2).

Ta có (2) tương đương:

cos(x + y + b− 2α)− cos(y + b− x) = cos(x + y + a− 2α)− cos(x + a− y)⇔ sin a− sin b = cos(y + b− x)− cos(x + a− y)

Tập san Toán học STAR EDUCATION

Page 14: Möclöc - geosiro.com · Sßdöngph÷ìngph¡p˜⁄isŁtrong chøngminhh…nhhåc Nguy„n Tăng Vũ (Giáo viên PTNK TP Hç Chí Minh) Bài toán hình håc phflng thưíng xoay

14 NGUYỄN TĂNG VŨ

Dễ thấy (1) tương đương:

cos(a+ x + y)− cos(a+ x − y) = cos(b+ x + y)− cos(b+ y − x)⇔ sin a− sin b = cos(b+ y − x)− cos(a+ x − y)

Từ (1), (2) và hai đẳng thức trên, ta có điều cần chứng minh.

Để kết thúc bài viết này, tác giả đưa ra cách giải cho bài toán thi chọn đội tuyển toántrường Phổ thông Năng khiếu năm 2017, đây là một bài toán khó, bằng chứng là cókhá ít bạn giải được câu a, còn câu b thì quá thử thách. Đáp án bài toán sử dụngphương pháp cực đối cực và khá ngắn gọn, tuy nhiên trong bài viết này tác giả trìnhbày bằng phương pháp biến đổi đại số, một phương pháp trực tiếp và không cần phảidựng yếu tố phụ. Cái hay của phương pháp đại số là khi tính toán cẩn thận và đúngđịnh hướng thì kết quả sẽ thật đẹp đúng như yêu cầu bài toán.

Ví dụ 10. Đường tròn (O) nội tiếp tứ giác ABC D và tiếp xúc với các cạnh AB, BC , C D, DAlần lượt tại E, F, G, H. Gọi I , J là trung điểm của AC , BD và IB, I D, JA, JC theo thứtự cắt EF, GH, HE, FG tại M , N , P,Q.

1. Chứng minh rằng I J , MN , PQ đồng quy (tại điểm S).

2. Các tia đối của các tia JA, IB, JC , I D lần lượt cắt (O) tại các điểm A′, B′, C ′, D′.Giả sử A′C ′, B′D′ lần lượt cắt PQ, MN tại U , V. Gọi K là hình chiếu của S trênUV. Chứng minh rằng ∠AKB = ∠CKD.

Lời giải.

1. • Trước hết, sử dụng định lý Menelaus trong các tam giác ABC và ADC , tathấy rằng các đường thẳng GH và EF cùng cắt AC tại một điểm, đặt là U .

Tập san Toán học STAR EDUCATION

Page 15: Möclöc - geosiro.com · Sßdöngph÷ìngph¡p˜⁄isŁtrong chøngminhh…nhhåc Nguy„n Tăng Vũ (Giáo viên PTNK TP Hç Chí Minh) Bài toán hình håc phflng thưíng xoay

NGUYỄN TĂNG VŨ

Tương tự, các đường thẳng EH, FG cùng cắt BD tại một điểm, đặt là V.Tiếp theo, áp dụng định lý Desargue cho hai tam giác AHP và CGQ ta cóGH; AC; PQ đồng quy tại U . Tương tự thì HE, MN , BD đồng quy tại V.

• Áp dụng định lý Menelaus cho tam giác AIU thì

DNIN·

IUAU·

AHDH= 1 nên

DNIN=

AUIU·

DHAH

.

• Gọi S′ = MN ∩ I J thì

IS′

JS′·

V JV D·

N DN I= 1 nên

IS′

JS′=

N IN D·

V DV J=

HAHD·

U IUA·

V DV J(1).

• Tương tự, ta cũng cóAPPJ=

V DV J·

AHHD

. Gọi S′′ là giao điểm của PQ và I J thì

IS′′

JS′′=

U IUS·

APPJ=

U IUA·

V DV J·

AHHD(2).

• Từ (1) và (2) suy ra S′ ≡ S′′. Vậy PQ, MN , I J đồng quy tại một điểm đặt làS. Ta có đpcm.

2. • Chứng minh S trùng O. Theo câu a thì

ISJS=

U IUA

.V DV J

.AHHD

(3).

Theo định lý đường thẳng Newton, ta có I , O, J thẳng hàng và

IOOJ=

C F + AEBE + DH

.

Ta cóAEBE·

BFC F·

UCUA= 1, suy ra C F =

AE · UCUA

. Suy ra

AE + C F = AE(1+UCUA) =

AE(UC + UA)UA

= 2AE · U I

UA.

Tương tự thì BF + DH =2DH · V J

V D. Khi đó

OIOJ=

2 · AE · U IUA

:2 · DH · V J

V D=

U IUA

.V DV J

.AHDH

(4).

Từ (3) và (4) suy ra S ≡ O.

• A′C ′, PQ, AC đồng quy tại U và B′D′; MN ; BD đồng quy tại V .Thật vậy, xét trong đường tròn (O), vì GH là đường đối cực của D và điqua U nên đường đối cực của U đi qua D. Tương tự thì đường đối cực củaU đi qua B nên BD là đường đối cực của U . Suy ra J(RU , C ′A′) = −1, suyra U , C ′, A′ thẳng hàng.

Do đó A′C ′, PQ, AC đồng quy. Tương tự với B′D′; MN ; BD.

Ta cũng chú ý rằng HF, GE, AC , BD đồng quy tại R.

Tập san Toán học STAR EDUCATION

Page 16: Möclöc - geosiro.com · Sßdöngph÷ìngph¡p˜⁄isŁtrong chøngminhh…nhhåc Nguy„n Tăng Vũ (Giáo viên PTNK TP Hç Chí Minh) Bài toán hình håc phflng thưíng xoay

16 NGUYỄN TĂNG VŨ

• Theo định lý Brokard thì OR⊥UV tại K .Ta có V (UR, V F) = −1, suy ra B(UR, EF) = −1, suy ra (U , R, A, C) = −1.Do đó K(UR, AC) = −1. Mà KU⊥KR nên KR là phân giác góc ∠AKC .Chứng minh tương tự, ta có KR là phân giác ∠BKD.

Vậy ∠AKB = ∠CKD. Bài toán được giải quyết hoàn toàn.

2. Bài tập

1. (Iran 1998) Cho tam giác ABC nhọn nội tiếp (O). Điểm D di chuyển trên cungBC không chứa A. Gọi I , J lần lượt là tâm nội tiếp các tam giác DAB, DAC . Chứngminh đường tròn ngoại tiếp tam giác DIJ luôn đi qua một điểm cố định khi Dthay đổi.

2. (4 IMO 2010) Cho P là một điểm nằm trong tam giác ABC(CA 6= CB). Cácđường thẳng AP, BP, C P lần lượt cắt đường tròn ngoại tiếp tam giác ABC (kíhiệu là w) tại K , L, M . Tiếp tuyến tại C của w cắt AB tại S. Chứng minh rằngnếu SC = SP thì MK = M L.

3. Cho AB và C D là 2 dây cung cắt nhau của (O). Đường tròn (w) tiếp xúc trong vớicung nhỏ BD của (O) tại T và tiếp xúc với AB, C D. P là một điểm di chuyển trêncung nhỏ BD của (O). Gọi I , J lần lượt là tâm nội tiếp các tam giác PAB, PC D.Chứng minh rằng P, I , J , T đồng viên.

4. Cho 2 tam giác ABC và DEF cùng nội tiếp một đường tròn (O). Gọi X Y Z là tamgiác tạo bởi 3 đường thẳng Simson của D, E, F đối với tam giác ABC và MN P làtam giác tạo bởi 3 đường thẳng Simson của A, B, C đối với tam giác DEF . Chứngminh X , Y, Z , M , N , P đồng viên.

5. (IMO 2013) Cho tam giác ABC nội tiếp (O) có A1 là tiếp điểm của đường trònbàng tiếp góc A lên cạnh BC . B1, C1 được xác định tương tự. Chứng minh nếutâm đường tròn ngoại tiếp tam giác A1B1C1 thuộc (O) thì ∆ABC vuông.

6. Cho tam giác ABC có E, F lần lượt là tiếp điểm của đường tròn nội tiếp (I) vớicạnh AC , AB. BM , CN là các đường cao của tam giác ABC . Chứng minh rằngtrực tâm ∆AEF cũng chính là tâm đường tròn nội tiếp ∆AMN .

7. (Vietnam TST 2008) Cho tam giác ABC nội tiếp (O) có phân giác trong AD, BE, C F .

Các điểm K , M , N lần lượt thuộc các đoạn AD, BE, C F sao choKAKD=

MBM E

=NCN F= k. Kí hiệu (X ) là đường tròn qua K , A và tiếp xúc với OA. Các đường tròn

(Y ), (Z) xác định tương tự.

(a) Với k =12

, chứng minh rằng (X ), (Y ), (Z) có 2 điểm chung và trọng tâm G

của tam giác ABC thuộc đường nối 2 điểm chung này.

(b) Tìm mọi k sao cho (X ), (Y ), (Z) có 2 điểm chung.

Tập san Toán học STAR EDUCATION

Page 17: Möclöc - geosiro.com · Sßdöngph÷ìngph¡p˜⁄isŁtrong chøngminhh…nhhåc Nguy„n Tăng Vũ (Giáo viên PTNK TP Hç Chí Minh) Bài toán hình håc phflng thưíng xoay

NGUYỄN TĂNG VŨ

8. (4a Vietnam TST 2014) Cho tam giác ABC có đường cao AD và P di động trênAD. Các đường thẳng PC và AB cắt nhau ở F , các đường thẳng PB và AC cắt

nhau ở E. Giả sử tứ giác AEDF nội tiếp, chứng minh rằng:PAPD= (tanB +

tanC).cotA/2

9. (3 USA TST 2013) Cho tam giác ABC vuông tại C có D là chân đường cao hạtừ C . Gọi X là một điểm thuộc đoạn C D. K thuộc đoạn AX sao cho BK = BC , Lthuộc đoạn BX sao cho AL = AC . Đường tròn ngoại tiếp tam giác DK L cắt ABtại điểm thứ 2 là T (T khác D). Chứng minh ∠AC T = ∠BC T .

10. Cho tam giác ABC nhọn nội tiếp (O). Đường tròn A-mixtilinear tiếp xúc với(O) tại A′. Tương tự định nghĩa B′, C ′. Gọi A′A là đường kính của đường trònA-mixtilinear, tương tự định nghĩa B, C. Chứng minh rằng AA, BB, CC đồngquy.

11. Cho tam giác ABC nội tiếp (O) có K là giao điểm của tiếp tuyến tại A và B và Llà giao điểm của tiếp tuyến tại A và C . Gọi P là giao điểm của CK và BL. Kẻ 3đường cao AD, BE, C F của tam giác ABC . Trung tuyến kẻ từ A của tam giác ABCcắt EF tại Q. Chứng minh D, P,Q thẳng hàng.

12. (mở rộng bài toán con bướm) Cho tứ giá ABC D nội tiếp (O). Gọi M là trungđiểm AB và N , P là 2 điểm thuộc AB đối xứng với nhau qua M . CN , C M cắt (O)lần lượt tại S, T (S, T khác C). DS, DT cắt AB lần lượt tại E, F . Chứng minh rằngM E = M F .

13. Cho tam giác ABC nội tiếp (O) có BC cố định, A thay đổi trên cung lớn BC . Trêncác tia BA, CA lấy M , N sao cho BM = CN . BN cắt C M tại K , KD cắt AC tại E.Chứng minh trung tuyến AI của ∆ADE luôn đi qua một điểm cố định.

14. Cho tam giác ABC nội tiếp (O) có BC cố định, A thay đổi và AH là đường cao,AK là phân giác trong. Gọi I là trung điểm AH, M là trung điểm AK . Điểm Nthuộc AK sao cho ∠AC M = ∠BCN . Chứng minh IN luôn đi qua một điểm cốđịnh.

15. Cho tam giác ABC có M là trung điểm BC . E, F di động trên AC , AB. M E cắtAB tại P, M F cắt AC tại Q. Gọi (w) là đường tròn ngoại tiếp tam giác AEF . AKlà đường kính của (w). Đường tròn ngoại tiếp tam giác APQ cắt (w) tại điểm Lkhác A. Chứng minh K L đi qua một điểm cố định.

16. Cho đường tròn (O). 6 đường tròn (O1), (O2), (O3), . . . (O6) tiếp xúc trong với(O) lần lượt tại A, B, C , D, E, F và đường tròn (Oi) tiếp xúc ngoài với đường tròn(Oi+1) với mọi i = 1, 2, . . . 6 ( O7 ≡ O1). Chứng minh AD, BE, C F đồng quy.

17. Cho ∆ABC có đường cao AD. Gọi (O1) là đường tròn tiếp xúc với tia DA, DBvà tiếp xúc trong với (O), (O2) là đường tròn tiếp xúc với tia DA, DC và tiếpxúc trong với (O). Tiếp tuyến chung trong khác AD của (O1), (O2) cắt BC tại M .Chứng minh M là trung điểm BC khi và chỉ khi AB + AC = 2BC .

18. Cho tam giác ABC nhọn. Dựng ra phía ngoài tam giác ABC các tam giác đềuABF và AC E. BE cắt C F tại T . Gọi S là điểm đẳng giác đối với T trong tam giácABC . Chứng minh rằng S là giao điểm chung của 3 đường tròn Apollonius ứngvới tam giác ABC .

Tập san Toán học STAR EDUCATION

Page 18: Möclöc - geosiro.com · Sßdöngph÷ìngph¡p˜⁄isŁtrong chøngminhh…nhhåc Nguy„n Tăng Vũ (Giáo viên PTNK TP Hç Chí Minh) Bài toán hình håc phflng thưíng xoay

Tài liệu tham khảo

[1] Hoàng Chúng, 1996, Hình học của tam giác NXB Giáo dục.

[2] Đỗ Thanh Sơn, 2010, Một số chuyên đề hình học phẳng bồi dưỡng học sinh giỏiTHPT NXB Giáo dục.

[3] Roger A.Johnson, 1964, Advanced Euclidean Geometry Dover Publications.

[4] Titu Adreescu và Cosmin Pohoata, 2016, Lemmas in Olympiad Geometry. XYZpress.

[5] Evan Chen, 2015, Euclidean Geometry mathematical Olympiads. MAA press, 2

[6] Hồ Quốc Đăng Hưng, 2016, Ứng dụng tỉ số đoạn thẳng và tỉ số lượng giác tronghình học

18

Page 19: Möclöc - geosiro.com · Sßdöngph÷ìngph¡p˜⁄isŁtrong chøngminhh…nhhåc Nguy„n Tăng Vũ (Giáo viên PTNK TP Hç Chí Minh) Bài toán hình håc phflng thưíng xoay

Bổ đề về số mũ đúng

Nguyễn Ngọc Duy(Giáo viên trường PTNK TP Hồ Chí Minh)

Bổ đề số mũ đúng của một số nguyên là một hướng tiếp cận khá mới đối với các bài toán sơcấp. Nó cung cấp một công cụ khá hữu hiệu để giải các phương trình Diophante hoặc các bàitoán chia hết liên quan đến số mũ. Trong bài viết này tôi sẽ cố gắng mang đến một cái nhìnthật sơ cấp và tự nhiên đến vấn đề, trang bị thêm kiến thức và kĩ năng cho các các em họcsinh để giải quyết các bài toán số học.Đặc biệt, ta sẽ dùng bổ đề số mũ đúng để giải quyết mộtsố trường hợp đặc biệt của định lí lớn Fermat.

1. Kiến thức cần nhớ

Định nghĩa 1. Cho (a, n) = 1. Kí hiệu cấp của a theo modulo n là ordn (a), là sốnguyên dương d nhỏ nhất thỏa ad ≡ 1 (modn).

Tính chất 1. Nếu x là số nguyên dương thỏa mãn ax ≡ 1 (modn) thì ordn (a) |x .

Định nghĩa 2. Cho p là số nguyên tố, x là số nguyên bất kì, kí hiệu vp (x) = n nếu xchia hết cho pn nhưng không chia hết cho pn+1 .

Tính chất 2. Với a, b là các số nguyên và n là số nguyên dương thì:

• vp (ab) = vp (a) + vp (b).

• Nếu p|a thì vp(a)> 0.

• vp (an) = nvp (a).

• vp (a+ b) ≥ min

vp (a) , vp (b)

. Đẳng thức xảy ra chẳng hạn khi vp(a) 6=vp(b).

• vp(gcd(a, b)) =min(vp(a), vp(b)) và vp(lcm(a, b)) =max(vp(a), vp(b)).

19

Page 20: Möclöc - geosiro.com · Sßdöngph÷ìngph¡p˜⁄isŁtrong chøngminhh…nhhåc Nguy„n Tăng Vũ (Giáo viên PTNK TP Hç Chí Minh) Bài toán hình håc phflng thưíng xoay

20 NGUYỄN NGỌC DUY

Định lý 1. Bổ đề số mũ đúng. Cho p là số nguyên tố lẻ; a, b không chia hết cho p

• Nếu a− b chia hết cho p thì vp (an − bn) = vp (a− b) + vp (n).

• Nếu a+ b chia hết cho p, n lẻ thì vp (an + bn) = vp (a+ b) + vp (n).

• Nếu a, b lẻ thì v2 (an − bn) = v2

x2−y2

2

+ v2 (n).

Chứng minh. i. Trước tiên, ta chứng minh: vp (ap − bp) = vp (a− b) + 1 (*). Ta có:

ap − bp = (a− b)

ap−1 + ap−2 b+ ...+ abp−2 + bp−1

.

Do a ≡ b (modp) nên ap−1 + ap−2 b+ ...+ abp−2 + bp−1 ≡ p.ap−1 ≡ 0 (modp).Suy ra : ap−1 + ap−2 b+ ...+ abp−2 + bp−1 chia hết cho p (1).Ta chứng minh tiếp

ap−1 + ap−2 b+ ...+ abp−2 + bp−1không chia hết cho p2.

Thật vậy, do a ≡ b (modp) nên a = b+ kp . Sử dụng khai triển nhị thức Newton ta có

ap−1 + ap−2 b+ · · ·+ bp−1

(p− 1) kpbp−2 + bp−1

+

(p− 2) kpbp−2 + bp−1

+

· · ·+

kpbp−2 + bp−1

+ bp−1

modp2

≡p (p− 1)

2kpbn−2 + p.bp−1 ≡ pbp−1

modp2

Theo giả thiết thì b không chia hết cho p nên pbp−1 không chia hết cho p2. Do đóap−1 + ap−2 b+ · · ·+ abp−2 + bp−1 cũng không chia hết cho p2 (2).Từ (1), (2) ta có: vp

ap−1 + ap−2 b+ · · ·+ abp−2 + bp−1

= 1.Vậy vp (ap − bp) = vp (a− b) + 1. Tương tự, ta cũng có: nếu m không chia hết cho pthì vp (am − bm) = vp (a− b) (**).Ta quay lại định lí, Đặt vp (n) = k⇒ n = pk.m, với (m, p) = 1. Áp dụng (*) và (**) tacó:

vp (an − bn) = vp

apk−1.mp−

bpk−1.mp

= vp

apk−1.m − bpk−1.m

+ 1= . . .= vp (am − bm) + k

= vp (a− b) + vp (n) .

Vậy ta đã chứng minh xong phần i. của định lí ii. Vì n lẻ nên thay b bởi −b trong i. tađược vp (an + bn) = vp (an − (−b)n) = vp (a+ b) + vp (n) Vậy ta đã chứng minh xongphần ii. của định lí iii. Tương tự cách làm trong i. ta cũng có kết quả iii. Như vậy ta đãchứng minh xong bổ đề số mũ đúng. Sau đây ta sẽ sử dụng bổ đề để giải quyết mộtbài toán thú vị.

2. Các bài toán áp dụng

Bài toán Fermat lớn: Cho n là số tự nhiên lớn hơn 2. Chứng minh rằng phương trìnhan + bn = cn không có nghiệm nguyên dương.

Tập san Toán học STAR EDUCATION

Page 21: Möclöc - geosiro.com · Sßdöngph÷ìngph¡p˜⁄isŁtrong chøngminhh…nhhåc Nguy„n Tăng Vũ (Giáo viên PTNK TP Hç Chí Minh) Bài toán hình håc phflng thưíng xoay

NGUYỄN NGỌC DUY

Bài Toán Fermat lớn là bài toán cực kì thú vị. Nó tồn tại gần bốn thế kỉ, kích thíchbiết bao nhà toán học thế giới. Bài toán cuối cùng được chứng minh bởi nhà toán họcAndrew Wiles vào năm 1993. Và người ta nói rằng sẽ không có phương pháp sơ cấpnào có thể chứng minh bài toán trên. Bài báo sẽ đề cập một trường hợp đặc biệt củabài toán: số c là số nguyên tố. Và chúng ta sẽ giải quyết thông qua bổ đề số mũ đúng.

Bài toán 1. Cho số nguyên lẻ n > 2, p là số nguyên tố. Chứng minh rằng phươngtrình an + bn = pn không có nghiệm nguyên dương.

Lời giải.Không mất tính tổng quát, giả sử phương trình có nghiệm a ≥ b .

1. Nếu a = 1⇒ b = 1, thế vào phương trình suy ra vô lí.

2. Nếu a = 2⇒ b = 1;2. Trường hợp (a, b) = (2, 2)⇒ p = 2(vô lí). Trường hợp(a, b) = (2,1)⇒ p = 3 , thế vào phương trình ta được 3n − 2n = 1 , cũng suy ravô lí.

Vậy bắt buộc a ≥ 3, mà pn > an⇒ p > 3 , nên p là số nguyên tố lẻ. Do n lẻ, ta có :

pn = an + bn = (a+ b)

an−1 − an−2 b+ · · · − abn−2 + bn−1

Suy ra p|a+ b (do a+ b > 1 ). Áp dụng bổ đề số mũ đúng cho p, ta có

vp

an−1 − an−2 b+ · · · − abn−2 + bn−1

= vp (an + bn)− vp (a+ b) = vp (n)

Mà an−1 − an−2 b+ · · · − abn−2 + bn−1 là lũy thừa của p nên ta có

an−1 − an−2 b+ · · · − abn−2 + bn−1

|n.

Do an−1−an−2 b+· · ·−abn−2+bn−1 = 12

an−1 + an−3(a− b)2 + · · ·+ bn−3(a− b)2 + bn−1

≥12

an−1 + bn−1

Vì a ≥ 3, n≥ 3 nên 12

an−1 + bn−1

> n nên không thể

an−1 − an−2 b+ · · · − abn−2 + bn−1

|n.

Vậy phương trình vô nghiệm khi p là số nguyên tố.

Bài toán 2. Cho số nguyên n > 2 có ước lẻ khác 1, p là số nguyên tố. Chứng minhrằng phương trình an + bn = pn không có nghiệm nguyên dương.

Lời giải. Gọi k > 1 là ước lẻ của n, giả sử n= km . Đặt x = am; y = bm. Phương trìnhtrên trở thành

x k + yk = pn.

Không mất tính tổng quát, giả sử x ≥ y . Tương tự bài toán 1 ta sẽ loại được cáctrường hợp tầm thường x = 1; x = 2 . Nên ta xét bài toán với trường hợp x , p ≥ 3. Dok lẻ, ta có pn = ak + bk = (a+ b)

ak−1 − ak−2 b+ · · · − abk−2 + bk−1

Suy ra p|b + a.Áp dụng bổ đề số mũ đúng cho p ta có

vp

ak−1 − ak−2 b+ · · · − abk−2 + bk−1

= vp

ak + bk

− vp (a+ b) = vp (k)

Mà ak−1 − ak−2 b+ · · · − abk−2 + bk−1 là lũy thừa của p nên ta có

ak−1 − ak−2 b+ · · · − abk−2 + bk−1

|k

Lập luận tương tự bài toán 1 ta cũng suy ra vô lí. Vậy phương trình vô nghiệm .

Tập san Toán học STAR EDUCATION

Page 22: Möclöc - geosiro.com · Sßdöngph÷ìngph¡p˜⁄isŁtrong chøngminhh…nhhåc Nguy„n Tăng Vũ (Giáo viên PTNK TP Hç Chí Minh) Bài toán hình håc phflng thưíng xoay

22 NGUYỄN NGỌC DUY

Bài toán 3. Cho số nguyên n = 2k, k > 1 , p là số nguyên tố. Chứng minh rằngphương trình an + bn = pn không có nghiệm nguyên dương.

Lời giải. Tương tự Bài toán 1, ta loại được các trường hợp tầm thường nên ta chỉ xétđối với trường hợp a, b có ít nhất một số lớn hơn 2, khi đó p > 3. Phương trình trởthành dạng

x4 + y4 = p2k

trong đó x , y có ít nhất một số lớn hơn 2 và (x , y) = 1. Do p lẻ nên x , y kháctính chẵn lẻ. Không mất tính tổng quát, giả sử x lẻ, y chẵn. Ta có y4 = p2k − x4 =

p2k−1+ x2

p2k−1 − x2

Do

p2k−1+ x2; p2k−1 − x2

= 2 nên

¨

p2k−1+ x2 = 2m1

2

p2k−1− x2 = 2n1

2

Suy ra¨

p2k−1= m1

2 + n12

x2 = m12 − n1

2

và y2 = 2m1n1. Ta thấy (m1; n1) = 1 vì nếu ngược lại thì m1 và m2 đều phải chia hếtcho p (vô lí) nên có các trường hợp sau

1. Nếu m1 = m22, n1 = 2n2

2 và (m2, n2) = 1 thì thế vào ta được

p2k−1= m2

4 + 4n24 =

m22 + 2m2n2 + 2n2

2

m22 − 2m2n2 + 2n2

2

m22 + 2m2n2 + 2n2

2, m22 − 2m2n2 + 2n2

2

= 1

nên

m22 − 2m2n2 + 2n2

2 = 1⇔ (m2 − n2)2 + n2

2 = 1⇔ m2 = n2 = 1.

Trường hợp này không thỏa.

2. Nếu m1 = 2m22, n1 = n2

2 và (m2, n2) = 1 thì cũng tương tự.

Vậy phương trình không có nghiệm nguyên dương.

Như vậy sử dụng bổ đề số mũ đúng ta đã chứng minh được một trường hợp đặc biệtcủa Định lí lớn Fermat. Sau đây, chúng ta sẽ sử dụng Bổ đề số mũ đúng để giải quyếtmột số bài toán khác.

Bài toán 4. Tìm bộ số nguyên dương (a, b, p) trong đó p là số nguyên tố thỏa

2a + pb = 15a.

Tập san Toán học STAR EDUCATION

Page 23: Möclöc - geosiro.com · Sßdöngph÷ìngph¡p˜⁄isŁtrong chøngminhh…nhhåc Nguy„n Tăng Vũ (Giáo viên PTNK TP Hç Chí Minh) Bài toán hình håc phflng thưíng xoay

NGUYỄN NGỌC DUY

Lời giải. Ta có ∀x , y ∈ Z; n ∈ N thì xn − yn...x + y nên pb = 15a − 2a...13⇒ p = 13.

Áp dụng bổ đề

b = v13

13b

= v13 (15a − 2a) = v13 (15− 2) + v13 (a)⇒ v13 (a) = b− 1⇒ a...13b−1

Mà a > 0 nên a ≥ 13b−1, suy ra

13b = 15a − 2a = (15− 2)

15a−1 + 15a−2.2+ · · ·+ 15.2a−2 + 2a−1

≥ (15− 2)

1513b−1−1 + 1513b−1−2.2+ · · ·+ 15.213b−1−2 + 213b−1−1

⇒ b = 1⇒ a = 1.

Vậy nghiệm bài toán là(a, b, p) = (1,1, 13).

Bài toán 5. Chứng minh rằng không tồn tại cặp số (a, n) nguyên dương, n > 2 , saocho (a+ 1)n − an là lũy thừa bậc dương của 5.

Lời giải. Giả sử tồn tại số nguyên dương m sao cho

(a+ 1)n − an = 5m.

Nhận xét: nếua hoặc a+ 1 chia hết cho 5 thì số còn lại cũng cũng chia hết cho 5 (vôlí). Nên cả hai số đều không chia hết cho 5. Ta xét các trường hợp:

1. Nếu a ≡ 1 (mod5)⇒ 0≡ (a+ 1)n − an ≡ 2n − 1 (mod5) . Suy ra 4|n.

2. Nếu a ≡ 2 (mod5)⇒ 0≡ (a+ 1)n − an ≡ 3n − 2n (mod5). Suy ra 2|n.

3. Nếu a ≡ 3 (mod5)⇒ 0≡ (a+ 1)n − an ≡ 4n − 3n (mod5). Suy ra 4|n.

Do đó, n luôn là số chẵn, đặt n= 2n1, (n1 ∈ N, n1 ≥ 2). Ta có

5m = (a+ 1)2n1 − a2n1 =

(a+ 1)2 − a2

(a+ 1)2(n1−1) + · · ·+ (a+ 1)2a2(n1−2) + a2(n1−1)

= (2a+ 1)

(a+ 1)2(n1−1) + (a+ 1)2(n1−2)a2 + ...+ (a+ 1)2a2(n1−2) + a2(n1−1)

.

Suy ra 5|2a+ 15 , áp dụng bổ đề số mũ đúng ta được

v5

(a+ 1)2(n1−1) + (a+ 1)2(n1−2)a2 + ...+ (a+ 1)2a2(n1−2) + a2(n1−1)

=v5

(a+ 1)2n1 − a2n1

− v5 (2a+ 1) = v5 (n1) .

Do (a+ 1)2(n1−1) + (a+ 1)2(n1−2)a2 + · · · + (a+ 1)2a2(n1−2) + a2(n1−1) là lũy thừa của 5nên

n1...

(a+ 1)2(n1−1) + (a+ 1)2(n1−2)a2 + ...+ (a+ 1)2a2(n1−2) + a2(n1−1)

(vô lí vì về phải gồm n1 số nguyên dương, n1 > 1 và a+1≥ 2). Vậy không tồn tại cặp

số (a, n) nguyên dương, n> 2 sao cho (a+ 1)n − an là lũy thừa bậc dương của 5.

Tập san Toán học STAR EDUCATION

Page 24: Möclöc - geosiro.com · Sßdöngph÷ìngph¡p˜⁄isŁtrong chøngminhh…nhhåc Nguy„n Tăng Vũ (Giáo viên PTNK TP Hç Chí Minh) Bài toán hình håc phflng thưíng xoay

24 NGUYỄN NGỌC DUY

Bài toán 6. Cho hai số nguyên a, n ≥ 2 sao cho tồn tại số nguyên dương k thỏan|(a− 1)k . Chứng minh rằng n là ước của 1+ a+ a2 + ...+ an−1 .

Lời giải. Giả sử p là ước nguyên tố bất kì của n . Theo giả thiết n|(a− 1)k nên p cũnglà ước của a − 1 . Do an − 1 = (a− 1)

1+ a+ a2 + · · ·+ an−1

nên áp dụng bổ đề sốmũ đúng ta có

vp

1+ a+ a2 + · · ·+ an−1

= vp (an − 1)− vp (a− 1) = vp (n) .

Do mọi ước nguyên tố p của n đều thỏa điều trên nên ta có

n|1+ a+ a2 + · · ·+ an−1.

Bài toán 7. (HSG Trung Quốc 2009) Tìm cặp số nguyên tố (p, q) thỏa pq|5p+5q (*).

Lời giải. Ta xét các trường hợp

1. p = q = 5 thỏa mãn bài toán.

2. Nếu có một số bằng 5, một số khác 5. Không mất tính tổng quát, giả sử p =5; q 6= 5. Ta có :

5q|55 + 5q⇔ q|54 + 5q−1⇔ q|54 + 1= 626

do 5q−1 ≡ 1 (mod q) nên suy ra q = 2 hoặc q = 313.

3. Nếu cả hai số p, q 6= 5 . Do 5p ≡ 5 (modp) , 5q ≡ 5 (mod q) nên

(∗)⇔

5p−1 + 1...q

5q−1 + 1...p⇒

52(p−1) − 1...q

52(q−1) − 1...p

Do 52(p−1)−1 chia hết cho q nhưng 5p−1−1 không chia hết cho q nên v2

ordq (5)

=

1 + v2 (p− 1). Do 5q−1 − 1 chia hết q nên q − 1...ordq (5) nên v2 (q− 1) ≥ 1 +

v2 (p− 1) . Tương tự khi xét chia hết cho p ta lại có v2 (p− 1) ≥ 1+ v2 (q− 1)(vô lí).

Vậy các cặp số thỏa mãn là (p, q) = (2,5) ; (5, 2) ; (5, 5) ; (5, 313) ; (313, 5) .

Bài toán 8. (HSG Brazil 2009) Cho hai số nguyên tố p, q sao cho q = 2p+1 . Chứngminh rằng tồn tại một số là bội của q có tổng các chữ số của nó trong hệ cơ số 10 nhỏhơn 4.

Lời giải. Do p, q đều là số nguyên tố nên q ≥ 5 . Nếu q = 5 thì ta chỉ cần chọn số 10thì thỏa yêu cầu bài toán. Nếu q > 5 , áp dụng Định lí Fermat nhỏ thì q|10q−1 − 1 =102p − 1= (10p − 1) (10p + 1) Suy ra q|10p + 1 hoặc q|10p − 1.

Tập san Toán học STAR EDUCATION

Page 25: Möclöc - geosiro.com · Sßdöngph÷ìngph¡p˜⁄isŁtrong chøngminhh…nhhåc Nguy„n Tăng Vũ (Giáo viên PTNK TP Hç Chí Minh) Bài toán hình håc phflng thưíng xoay

NGUYỄN NGỌC DUY

1. Nếu q|10p + 1 thì số a = 10p + 1 là số thỏa yêu cầu đề bài.

2. Nếu q|10p − 1. Do p là số nguyên tố và q không là ước của 10− 1(do q > 5 )nên p cũng chính là ordq (10). Do đó 10;102; . . . ; 10p sẽ có số dư khác nhau khichia cho q. Ta sẽ có các trường hợp

• Nếu tồn tại 1 ≤ k ≤ p mà 10k ≡ p (mod q) thì 2.10k + 1 ≡ 2p + 1 ≡0 (mod q). Khi đó số a = 2.10k + 1 là số thỏa yêu cầu đề bài.

• Nếu tồn tại 1 ≤ k ≤ p mà 10k ≡ 2p (mod q) thì 10k + 1 ≡ 2p + 1 ≡0 (mod q). Khi đó số a = 10k + 1 là số thỏa yêu cầu đề bài.

• Nếu không tồn tại 1 ≤ k ≤ p mà 10k có số dư là p hay 2p khi chia cho q.Thì ta sẽ chia các số dư còn lại của q thành p bộ

(1;2p− 1) , (2; 2p− 2) , . . . , (p− 1; p+ 1)

(tổng 2 phần tử của một bộ bằng 2p) . Do tập số dư khi chia cho q của tập

10; 102; . . . ; 10p

có p phần tử nên Theo nguyên lí Dirichlet sẽ có ít nhấthai số 10k và 10l thuộc cùng một bộ. Khi đó số a = 10k + 10l + 1 sẽ chiahết cho q là số thỏa yêu cầu đề bài.

Bài toán 9. (IMO Shortlist 1997) Cho b, m, n là các số nguyên dương thỏam >1; m 6= n. Biết bm − 1và bn − 1 có cùng tập hợp các ước nguyên tố. Chứng minhb+ 1 là lũy thừa của 2.

Lời giải. Theo đề, gọi p là ước nguyên tố bất kì của bm − 1và bn − 1. Ta có kết quảquen thuộc:

(bm − 1, bn − 1) = b(m,n) − 1,

đặt α= (m, n) nên p|bα−1. Suy ra tồn tại k, l ∈ N∗ thỏa m= αk; n= αl. Đặt a = bα

, từ giả thiết suy ra mọi ước nguyên tố của ak − 1 và al − 1 đều là ước của a− 1 . Nóicách khác, tập hợp các ước nguyên tố của ak − 1, al − 1 và a − 1 là trùng nhau. Dom 6= n suy ra tồn tại một số k hoặc l lớn hơn 1. Giả sử số đó là k. Ta chứng minh a+1là lũy thừa của 2. Thật vậy:

1. Nếu k là số chẵn, đặt k = 2β .k′(k′ là số lẻ). Ta có:

ak − 1=

ak′ − 1

ak′ + 1

a2k′ + 1

...

a2β−1k′ + 1

.

Do đó mọi ước nguyên tố q của ak′ + 1 cũng là ước của a − 1 Mà ak′ + 1...a + 1,

(a+ 1; a− 1) = 1 hoặc 2. Suy ra 2...q⇒ q = 2 nên ak′ + 1 là lũy thừa của 2. Suy

ra a+ 1 cũng là lũy thừa của 2.

2. Nếu k là số lẻ, ta có ak − 1 = (a− 1)

ak−1 + ak−2 + ...+ a+ 1

Gọi q là ướcnguyên tố bất kì của ak−1+ ak−2+ ...+1. Do ak−1+ ak−2+ ...+ a+1 là số lẻ nên,nên q cũng lẻ và là ước của ak−1 . Do đó q cũng là ước của a−1 . Áp dụng bổ đềsố mũ đúng của q ta có vq

ak−1 + ak−2 + ...+ 1

= vq

ak − 1

−vq (a− 1) = vq (k)

Suy ra k...

ak−1 + ak−2 + ...+ 1

(vô lí vì vế phải có k số nguyên dương, a > 1 ).

Tập san Toán học STAR EDUCATION

Page 26: Möclöc - geosiro.com · Sßdöngph÷ìngph¡p˜⁄isŁtrong chøngminhh…nhhåc Nguy„n Tăng Vũ (Giáo viên PTNK TP Hç Chí Minh) Bài toán hình håc phflng thưíng xoay

26 NGUYỄN NGỌC DUY

Vậy a+ 1= bα + 1 là lũy thừa của 2. Vì bα + 1 là lũy thừa của 2 nên nếu α là số chẵnthì bα + 1 =

bα′2+ 1 hoặc là số lẻ hoặc chia 4 dư 2 nên chỉ có một trường hợp thỏa

là b = 1 . Còn nếu α là số lẻ thì bα + 1 = (b+ 1)

bα−1 + bα−2 + ...+ b+ 1

nên b+ 1cũng là lũy thừa của 2.

Bài toán 10. (IMO Shortlist 1999) Tìm các số nguyên dương n, p trong đó p nguyêntố thỏa np−1|(p− 1)n + 1.

Lời giải. Ta xét các trường hợp sau

1. Nếu p = 2⇒ n|2⇒ n= 1;2 (thỏa).

2. Nếu p > 2 , suy ra p lẻ nên (p− 1)n + 1 lẻ ⇒ n lẻ Gọi q là ước nguyên tốnhỏ nhất của n ⇒ q|np−1|(p− 1)n + 1 ⇒ q|(p− 1)2n − 1 Mà : q|(p− 1)q−1 −1 ⇒ q|(p− 1)(2n,q−1) − 1 Do n lẻ và q là ước nguyên tố nhỏ nhất của n nên(2n; q− 1) = 2 . Suy ra q|(p− 1)2 − 1 = (p− 2) p ⇒ q|p − 2 hoặc q = p. Ta lạicó các trường hợp nhỏ

(a) Nếu q|p− 2⇒ 0≡ (p− 1)n + 1≡ 1+ 1≡ 2 (mod q)⇒ q = 2 (vô lí vì q lẻ)

(b) Nếu q = p . Áp dụng bổ đề số mũ đúng cơ số q ta có (p− 1) vp (n) =vp

np−1

≤ vp [(p− 1)n + 1] = vp (p− 1+ 1) + vp (n) = 1+ vp (n) Suy ra :(p− 2) vp (n)≤ 1⇒ p = 3 và vp (n) = 1.

Đến đây, bài toán trở thành : Tìm n để n2|2n + 1.

Nhận xét n = 1 thỏa yêu cầu bài toán nên ta xét n > 1. Suy ra n là số lẻ, gọi r làước nguyên tố nhỏ nhất của n. Suy ra r|2n + 1 |22n − 1, mà r|2r−1 − 1 nên suy rar|2(2n;r−1) − 1.

Do n là số lẻ và r là ước nguyên tố nhỏ nhất của n nên (2n; r − 1) = 2 nên r = 3. Tacó đánh giá sau

2v3 (n)≤ v3 (4n − 1) = v3 (4− 1) + v3 (n)⇒ v3 (n)≤ 1⇒ v3 (n) = 1.

Suy ra n= 3.m, (m, n) = 1. Thế vào đề bài, ta được

m2|8m + 1|82m − 1.

Nếu m > 1 , tương tự ta gọi s là ước nguyên tố nhỏ nhất của m. Suy ra m là ước của82 − 1= 63. Do đó s = 7, điều này vô lí vì 8m + 1 chia 7 dư 2. Suy ra m= 1⇒ n= 3.

Vậy (n, p) = (1,2) ; (2,2) ; (3; 3) .

Tập san Toán học STAR EDUCATION

Page 27: Möclöc - geosiro.com · Sßdöngph÷ìngph¡p˜⁄isŁtrong chøngminhh…nhhåc Nguy„n Tăng Vũ (Giáo viên PTNK TP Hç Chí Minh) Bài toán hình håc phflng thưíng xoay

Đa thức bất khả quy

Vương Trung Dũng(Giáo viên PTNK TP Hồ Chí Minh)

1. Giới thiệu sơ lược

Đa thức bất khả qui là một vấn đề kinh điển trong đa thức nói riêng và trong toán họcnói chung. Các bài toán về đa thức bất khả qui cũng thường xuyên xuất hiện trongcác kì thi Olympic về toán. Người ta quan tâm nhiều nhất về tính bất khả qui của mộtđa thức trên vành Z[x] và Q[x]. Có nhiều cách để kiểm tra tính bất khả qui của mộtđa thức loại này chẳng hạn như dùng trực tiếp định nghĩa hoặc dùng các tiêu chuẩnnhư tiêu chuẩn Eisenstein, tiêu chuẩn Perron, tiêu chuẩn Cohn, tiêu chuẩn Dumas...tuy nhiên bài viết này chỉ đề cập đến hai phương pháp thường được sử dụng nhất làsử dụng trực tiếp định nghĩa và tiêu chuẩn Eisenstein và các dạng mở rộng của nócùng với đó là một kĩ thuật tối quan trọng là rút gọn theo một modulo nguyên tố p.Các tiêu chuẩn khác hi vọng sẽ có dịp trình bày trong một bài viết khác.Trong tài liệu này ta qui ước Zp = Z/pZ và K là một trong các tập Z, Q, R, Zp. Khiđó, K[x] ( tương ứng K[x , y]) là các vành đa thức một biến (tương ứng 2 biến) cóhệ số trong K.

Định nghĩa 1. Đa thức P(x) trong vành K[x] được gọi là khả qui trên K nếu P(x) =f (x).g(x) trong đó f (x), g(x) là các đa thức không khả nghịch trong K[x]. Đa thứcP(x) được gọi là bất khả qui nếu P(x) không khả nghịch và không khả qui.Nói riêng, khi K là một trường thì một đa thức P(x) ∈K[x] có bậc dương được gọi làkhả qui trên K nếu có thể phân tích được thành tích hai đa thức có bậc dương trongK[x], ngược lại P(x) được gọi là bất khả qui trên K.

Định lý 1 (Gauss). Các vành đa thức

• R[x], C[x], Q[x], Zp[x]

• Z[x], Z[x , y], Q[x , y]...

là có sự phân tích duy nhất thành các nhân tử bất khả qui và sự phân tích này là duynhất. Nói riêng các khái niệm về đa thức bất khả qui, ước chung lớn nhất, bội chungnhỏ nhất vẫn còn đúng trên các vành này.

Lưu ý rằng trong trường hợp 1 ở trên là các đa thức có hệ số trên trường nên trên đóthuật toán Euclid hay định lí Bezout vẫn còn đúng nhưng trường hợp 2 thì không.

27

Page 28: Möclöc - geosiro.com · Sßdöngph÷ìngph¡p˜⁄isŁtrong chøngminhh…nhhåc Nguy„n Tăng Vũ (Giáo viên PTNK TP Hç Chí Minh) Bài toán hình håc phflng thưíng xoay

28 VƯƠNG TRUNG DŨNG

2. Tính bất khả qui trên C[x] và R[x]

Định lý 2. Mọi đa thức có bậc lớn hơn 1 đều khả qui trên C[x]

Chứng minh: Giả sử degP > 1. Ta thừa nhận định lí cơ bản của đại số "Mọi đa thứcP(x) ∈ C[x] có bậc lớn hơn 1 đều có ít nhất một nghiệm trên C". Khi đó P(x) cónghiệm x0 ∈ C nên theo Định lí Bezout

P(x) = (x − x0)Q(x),

trong đó deg ≥ 1 nên P(x) khả qui trên C[x].

Định lý 3. Mọi đa thức có hệ số thực bậc lớn hơn 2 đều khả qui trên R[x]. Nói riêngmột đa thức là bất khả qui trên R[x] khi và chỉ khi nó là đa thức bậc nhất hoặc bậc2 vô nghiệm.

Chứng minh:Giả sử P ∈ R[x] và degP > 2. Nếu deg P lẻ thì P có ít nhất một nghiệm thực nên nókhả qui. Nếu deg P chẵn thì P có một nghiệm phức α, khi đó α cũng là nghiệm của Pvà do đó P(x) = (x −α)(x −α)Q(x) là khả qui.

3. Tính bất khả qui trên Z[x] và Q[x]

Qua Định lí 2.1 và Định lí 2.2 ta thấy nếu K = C, R thì tính bất khả quy là đơn giảnnên ta quan tâm đến trường hợp K= Z, Q. Thật may mắn là bổ đề Gauss mà ta trìnhbày sau đây sẽ cho ta một sự tương ứng về tính bất khả qui của một đa thức hệ sốnguyên trên Z[x] và Q[x].

Định nghĩa 2. Cho đa thức P(x) = an xn+ an−1 xn−1+ ...+ a1 x + a0 ∈ Z[x], đa thứcP được gọi là nguyên bản nếu gcd(an, ..., a0) = 1

Tính chất 1. Tích của hai đa thức nguyên bản là một đa thức nguyên bản.

Tính chất 2. Mọi đa thức P ∈Q[x] đều viết được dưới dạng P = cP0(x), trong đó P0 làmột đa thức nguyên bản và c0 ∈Q.

Định lý 4. (Bổ đề Gauss) Một đa thức hệ số nguyên, có bậc dương bất khả qui trongQ[x] khi và chỉ khi nó bất khả qui trong Z[x].

Chứng minh:Hiển nhiên nếu P(x) bất khả qui trên Q[x] sẽ bất khả qui trên Z[x]. Ngược lại giảsử P(x) bất khả qui trên Z[x] mà P(x) = P1(x)P2(x), với P1, P2 ∈ Q[x] và 1 ≤deg P1, degP2 ≤ deg P. Khi đó ta viết lại P1 =

a1

b1Q1(x), P2 =

a2

b2Q2(x), với (ai, bi) = 1

và Q i nguyên bản, i ∈ 1,2.

Tập san Toán học STAR EDUCATION

Page 29: Möclöc - geosiro.com · Sßdöngph÷ìngph¡p˜⁄isŁtrong chøngminhh…nhhåc Nguy„n Tăng Vũ (Giáo viên PTNK TP Hç Chí Minh) Bài toán hình håc phflng thưíng xoay

VƯƠNG TRUNG DŨNG

Suy ra P(x) =a1a2

b1 b2Q1(x)Q2(x) =

pq

Q1(x)Q2(x), trong đó p = a1 b1, q = a2 b2 và

(p, q) = 1.Do P ∈ Z[x] nên các hệ số của Q1(x)Q2(x) phải chia hết cho q điều này trái với tínhnguyên bản của Q1(x)Q2(x). Từ đó ta có điều phải chứng minh.

Định lý 5. Cho P(x) = an xn + an−1 xn−1 + ...+ a1 x + a0 ∈ Z[x]. Giả sử P có nghiệm

hữu tỉ x =pq

với (p, q) = 1. Khi đó, p là ước của a0 còn q là ước của an. Nói riêng,

mọi nghiệm hữu tỉ của một đa thức monic (đơn khởi, hệ số của bậc cao nhất bằng ±1)với hệ số nguyên đều là nghiệm nguyên.

Chứng minh:

Giả sử P(x) có nghiệm hữu tỉpq

, với (p, q) = 1. Khi đó

an(pq)n + ...+ a1.

pq+ a0 = 0,

qui đồng mẫu số ta đượcanpn + ...+ a0qn = 0.

Vì vế phải chia hết cho p nên vế trái chia hết cho p, từ đó suy ra a0qn chia hết cho p,lại có (qn, p) = 1 nên a0 chia hết cho p. Lập luận tương tự ta được an chia hết cho q.

Định lý 6. Cho P ∈ Q[x] có bậc 2 hoặc 3. Khi đó, P(x) là bất khả qui khi và khi khiP(x) không có nghiệm hữu tỉ.

Chứng minh:Hiển nhiên nếu P có nghiệm hữu tỉ thì nó khả qui. Đảo lại, nếu P khả qui thì P phântích được thành tích của hai đa thức hữu tỉ. Điều kiện bậc của P bằng 2 hoặc 3 chứngtỏ một trong hai nhân tử trên phải có bậc 1. Từ đó suy ra f có nghiệm hữu tỉ.Lưu ý: Định lí trên vẫn còn đúng nếu ta thay Q bởi một trường K bất kì. Tức là, đathức f ∈ K[x] với bậc bằng 2 hoặc 3 là bất khả qui nếu và chỉ nếu nó không cónghiệm trong K.Dưới đây là một số ví dụ

Ví dụ 1. (Định lí Schur) Cho các số nguyên phân biệt a1, a2, ..., an. Khi đó đa thức

1. f (x) = (x − a1)(x − a2)...(x − an)− 1 là bất khả qui trên Q[x].

2. f (x) = (x − a1)(x − a2)...(x − an) + 1 là bất khả qui trên Q[x] ngoại trừ cáctrường hợp

(b1) (x − a)(x − a− 2) + 1= (x − a− 1)2,

(b2) (x−a)(x−a−1)(x−a−2)(x−a−3)+1= [(x−a−1)(x−a−2)+1]2.

Lời giải. 1. Giả sử (x−a1)(x−a2)...(x−an)−1= g(x)h(x), với 1≤ deg f , deg g ≤n− 1 và g, h ∈ Z[x].

Tập san Toán học STAR EDUCATION

Page 30: Möclöc - geosiro.com · Sßdöngph÷ìngph¡p˜⁄isŁtrong chøngminhh…nhhåc Nguy„n Tăng Vũ (Giáo viên PTNK TP Hç Chí Minh) Bài toán hình håc phflng thưíng xoay

30 VƯƠNG TRUNG DŨNG

Ta có g(ai)h(ai) = −1 với mọi i, từ đó do g(ai), h(ai) là các số nguyên nên(g +h)(ai) = 0, với mọi i = 1, 2, ..., n. Nhưng vì deg (g +h)≤ n−1 triệt tiêu tạin giá trị phân biệt nên g ≡ −h. Từ đó

(x − a1)(x − a2)...(x − an)− 1= −(g(x))2.

Đẳng thức trên là vô lí vì hệ số cao nhất ở hai vế trái dấu.

2. Lập luận hoàn toàn như trên, giả sử f (x) là khả qui, bằng một phép đổi biếnđơn giản ta hoàn toàn có thể viết lại f dưới dạng

f (x) = x(x − a1)(x − a2)...(x − an−1) + 1= g(x).h(x),

trong đó 0< a1 < a2 < ...< an−1 và 1≤ g, h ∈ Z, deg(g), deg(h)≤ n− 1.

Từ đẳng thức g(ai)h(ai) = 1 ta suy ra g(ai) = h(ai) = ±1 với mọi i và đẳngthức này xảy ra tại n giá trị phân biệt. Điều đó dẫn dến g(x) = h(x) và ta có

f (x) = g2(x). Nói cách khác, deg( f ) = n là một số chẵn. Khi đó f (12) =

12(12−

a1)...(12− an−1)+1= 1−

12n(2a1−1) . . . (2an−1−1)≤ 1−

12n

1.3 . . . (2n−3)< 0

với mọi n≥ 6 (vô lí). Như vậy n= 2 hoặc n= 4.

(a) Nếu n = 2 thì f (12) = 1 −

14(2a1 − 1) ⇒ a1 ≤

52

và do đó a1 = 1,2.

Giá trị a1 cho ta f (x) = x(x − 1) + 1 là bất khả qui và a1 = 2 cho taf (x) = x(x − 2) + 1= (x − 1)2 là khả qui.

(b) Nếu n = 4 thì f (12) = 1 −

116(2a1 − 1)(2a2 − 1)(2a3 − 1) ⇒ 0 ≤

116(2 −

1)(3 − 1)(2a3 − 1) ⇒ a3 ≤196

. Xét trường hợp a1 = 1, a2 = 2, a3 = 3 ta

đượcf (x) = x(x − 1)(x − 2)(x − 3) + 1= (x2 − 3x + 1)2

là khả qui.

Bài toán được chứng minh xong.

Ví dụ 2. Cho a1, a2, ..., an là các số nguyên dương phân biệt. Chứng minh rằng đathức

P(x) = (x − a1)2(x − a2)

2...(x − an)2 + 1

là bất khả qui trên Z.

Lời giải. Giả sử P(x) là khả qui, tức tồn tại hai đa thức G(x), H(x) ∈ Z[x] có bậckhông bé hơn 1 sao cho P(x) = G(x).H(x). Ta có P(ai) = G(ai).H(ai) với i = 1, 2, ..., nnên G(ai) = H(ai) = ±1. Ta xét các trường hợp

1. Nếu deg G = deg H thì deg(G−H)≤ n−1⇒ G ≡ H. Từ đó P(x) = (G(x))2⇔1=

G(x)− (x − a1)...(x − an)

G(x)+ (x − a1)...(x − an)

, vô lí vì bậc vế phảiluôn không nhỏ thua 1.

Tập san Toán học STAR EDUCATION

Page 31: Möclöc - geosiro.com · Sßdöngph÷ìngph¡p˜⁄isŁtrong chøngminhh…nhhåc Nguy„n Tăng Vũ (Giáo viên PTNK TP Hç Chí Minh) Bài toán hình håc phflng thưíng xoay

VƯƠNG TRUNG DŨNG

2. Nếu degH < degG thì degH < n mà H(ai) = ±1, i = 1,2, ..., n dẫn đến H là đathức hằng, vô lí.

Vậy P(x) bất khả qui.

4. Rút gọn modulo p nguyên tố

Kĩ thuật rút gọn modulo p nguyên tố là một kĩ thuật tối quan trọng trong việc chứngminh một đa thức là bất khả qui trên Z. Nó đưa các hệ số từ một trường vô hạn cácphần tử về một trường hữu hạn các phần từ, từ đó các tính toán của ta có thể đượcđơn giản hơn.

Định nghĩa 3. Cho P(x) =n∑

i=0ai x i ∈ Z[x], an 6= 0 và p là số nguyên tố. Giả sử p

không phải là ước của gcd(a1, a2, ..., an). Ta kí hiệu P là đa thức nhận được từ Pbằng cách rút gọn các hệ số theo modulo p (lúc này P(x) ∈ Zp[x]). Khi đó ta gọi Plà đa thức rút gọn theo modulo p của P.

Từ định nghĩa trên ta có sự kiện sau là hiển nhiên

P +Q = P +Q

PQ = P. Q

Định nghĩa 4. Nếu đa thức rút gọn modulo p của P bất khả qui thì ta nói đa thức Pbất khả qui mod p.

Định lý 7. Với mỗi P(x) ∈ Z[x], tồn tại các đa thức P1(x), P2(x), ..., Pk(x) ∈ Zp[x]sao cho

P(x) = P1(x).P2(x)...Pk(x),

sự phân tích này là duy nhất theo modulo p.

Định lý 8. Cho P(x) =n∑

i=0ai x i ∈ Z[x], an 6= 0 và p không là ước của an. Khi đó, nếu

P(x) là bất khả qui mod p thì P(x) là bất khả qui. Điều ngược lại của định lí nóichung không đúng.

Chứng minh:Giả sử P(x) = an xn + an−1 xn−1 + ...+ a1 x + a0 ∈ Z[x] và p không là ước của an. Giảsử P(x) = f (x).g(x) với f , g ∈ Z[x] với deg f , g ≥ 1. Khi đó P = f .g. Do p không làước của an nên bậc của các đa thức P, f , g không thay đổi sau khi rút gọn theo modulop. Điều này chứng tỏ P khả qui theo modulo p, vô lí. Ta có điều phải chứng minh.Ngược lại dễ thấy đa thức P(x) = x4+1 bất khả qui trên Z[x] nhưng khả qui modulop với mọi số nguyên tố p.

Tập san Toán học STAR EDUCATION

Page 32: Möclöc - geosiro.com · Sßdöngph÷ìngph¡p˜⁄isŁtrong chøngminhh…nhhåc Nguy„n Tăng Vũ (Giáo viên PTNK TP Hç Chí Minh) Bài toán hình håc phflng thưíng xoay

32 VƯƠNG TRUNG DŨNG

Ví dụ 3. Chứng minh đa thức P(x) = x5 + 4x4 + 2x3 + 5x2 − 7 là bất khả qui.Lời giải:

• Rút gọn theo modulo 2 ta được P(x) = x5 + x2 + 1.

• Giả sử P(x) = f (x).g(x), với f , g ∈ Z2.

• Nếu deg f = 1 hoặc deg g = 1 dễ thấy là vô lí vì P không có nghiệm trong Z2.

• Suy ra P(x) = (x2 + ax + b)(x3 + cx2 + d x + e), với a, b, c, d, e ∈ Z2. Đồngnhất hệ số hai vế ta được điều vô lí. Từ đó suy ra điều phải chứng minh.

Ta có thể liệt kê ra các đa thức bất khả qui modulo 2 trong một số trường hợp bậc nhỏnhư sau

• Trường hợp n= 1 gồm các đa thức: x , x + 1.

• Trường hợp n= 2 chỉ gồm một đa thức: 1+ x + x2.

• Trường hợp n= 3 gồm các đa thức: 1+ x + x3, 1+ x2 + x3.

• Trường hợp n= 4 gồm các đa thức: 1+ x + x4, 1+ x + x2 + x3 + x4.

• Trường hợp n= 5 gồm các đa thức: 1+ x + x2+ x4+ x5, 1+ x + x3+ x4+ x5, 1+x2 + x3 + x4 + x5, 1+ x + x2 + x3 + x5, 1+ x3 + x5, 1+ x2 + x5.

5. Tiêu chuẩn Eisenstein và một số dạng mở rộng

Khi kiểm tra đa tính bất khả qui của một đa thức trên Z[x] tiêu chuẩn Eisenstein cungcấp cho ta một công cụ hiệu quả.

Định lý 9. (Tiêu chuẩn Eisenstein) Cho đa thức P(x) =n∑

i=0ai x

i ∈ Z[x], an 6= 0. Khi

đó nếu tồn tại số nguyên tố p thỏa đồng thời các điều kiện

• p không là ước của an;

• p là ước của ai với mọi i ∈ 1,2, ..., n− 1;

• p2 không là ước của a0.

Khi đó P(x) là đa thức bất khả qui trên Q[x].

Chứng minh:Có rất nhiều cách chứng minh cho tiêu chuẩn Eisenstein, ở đây ta sẽ trình bày chứngminh bằng cách rút gọn theo một modulo p nguyên tố bất kì.Giả sử f khả qui, tức f (x) = g(x).h(x), với f , g ∈ Z[x] và deg f , g ≥ 1. Rút gọn theomodulo p nguyên tố ta được đẳng thức trong Zp[x] dưới dạng

f = g.h.

Tập san Toán học STAR EDUCATION

Page 33: Möclöc - geosiro.com · Sßdöngph÷ìngph¡p˜⁄isŁtrong chøngminhh…nhhåc Nguy„n Tăng Vũ (Giáo viên PTNK TP Hç Chí Minh) Bài toán hình håc phflng thưíng xoay

VƯƠNG TRUNG DŨNG

Từ điều kiện p là ước của a0, ..., an−1 nhưng không là ước của an ta suy ra f = an xn.

Từ đó suy ra g = bk x k, h = bm xm. Điều này có nghĩa là b0 ≡ ... ≡ bk−1 ≡ c0 ≡ ... ≡cm−1 ≡ 0 mod(p). Nhưng khi đó a0 = b0c0 ≡ 0 (mod p2) (vô lí). Từ đó ta có điều phảichứng minh.

Ví dụ 4. Chứng minh đa thức P(x) = x4 − x2 + 2x + 1 bất khả qui trên Z.Lời giải:Đặt Q(x) = P(x + 1) = x4 + 3x3 + 3x2 + 3x + 3. Khi đó theo tiêu chuẩn Eisensteinvới p = 3 ta có điều phải chứng minh.

Định lý 10. (Dạng mở rộng thứ nhất của tiêu chuẩn Eisenstein) Cho f (x) = an xn +an−1 xn−1 + ...+ a1 x + a0 ∈ Z[x]. Giả sử tồn tại số nguyên tố p thỏa mãn với một sốtự nhiên k ≤ n nào đó mà

• p không là ước của ak;

• p là ước của a0, ..., ak−1;

• p2 không là ước của a0.

Thế thì f (x) có một nhân tử bất khả qui bậc ≥ k ( và do đó nếu không bất khả qui sẽcó một nhân tử bậc ≤ n− k)

Chứng minh: Bạn đọc có thể tự chứng minh như trong trường hợp nguyên bản củađịnh lí.

Ví dụ 5. Chứng minh đa thức f (x) = x101 + 101x100 + 102 là bất khả qui.Lời giải:Áp dụng tiêu chuẩn Eisenstein mở rộng cho p = 2, k = 100 ta thấy, nếu f là khả quithì nó phải có một nhân tử bậc 1 và do đó f phải có nghiệm hữu tỉ. Nói riêng vì hệsố bậc cao nhất bằng 1 nên nghiệm hữu tỉ này phải là nghiệm nguyên. Dễ thấy điềunày là không xảy ra. bài toán được chứng minh xong.

Định lý 11. (Dạng mở rộng thứ hai của tiêu chuẩn Eisenstein) Cho f (x) = an xn +an−1 xn−1 + ...+ a1 x + a0 ∈ Z[x]. Giả sử tồn tại số nguyên tố p thỏa mãn với một sốtự nhiên k ≤ n nào đó mà

• p không là ước của an;

• p là ước của a0, ..., an−1;

• p2 không là ước của ak.

Thế thì, hoặc f (x) là bất khả qui, hoặc f có một nhân tử bất khả qui bậc ≤ k.

Chứng minh: Tương tự như trên, chứng minh được dành cho bạn đọc.

Tập san Toán học STAR EDUCATION

Page 34: Möclöc - geosiro.com · Sßdöngph÷ìngph¡p˜⁄isŁtrong chøngminhh…nhhåc Nguy„n Tăng Vũ (Giáo viên PTNK TP Hç Chí Minh) Bài toán hình håc phflng thưíng xoay

34 VƯƠNG TRUNG DŨNG

6. Các bài toán áp dụng

Bài toán 1. (IMO 1993) Cho số tự nhiên n > 1. Chứng minh đa thức f (x) = xn +5xn−1 + 3 là bất khả qui trên Z[x].

Lời giải. Áp dụng dạng mở rộng thứ nhất của tiêu chuẩn Eisenstein với p = 3, k = n−1ta có điều phải chứng minh.

Bài toán 2. (China TST 1994) Cho số tự nhiên n ≥ 3 và hai số nguyên tố p, q phânbiệt. Tìm tất cả các số nguyên a sao cho đa thức P(x) = xn + axn−1 + pq bất khả quitrên Z.

Lời giải. Nếu p|a hoặc q|a thì theo tiêu chuẩn Eisenstein P(x) là bất khả qui. Xéttrường hợp p, q đều không là ước của a. Giả sử P khả qui, áp dụng dạng mở rộng thứnhất của tiêu chuẩn Eisenstein ta suy ra P(x) phải có nhân tử bậc 1 và do đó P cómột nghiệm nguyên x0.

Từ đó suy ra pq = −xn−10 (x0 + a). Vì n≥ 3 nên pq

... x20 nhưng vì p 6= q nên x0 = ±1.

Vì 1 + a + pq = 0 và (−1)n + a(−1)n−1 + pq = 0 nên a = −1 − pq hoặc a = 1 +(−1)npq.

Bài toán 3. (Rumani TST 1998) Chứng minh rằng đa thức P(x) = (x2 + x)2n+ 1 là

bất khả qui với mọi số tự nhiên n.

Lời giải. Bằng qui nạp ta chỉ ra rút gọn modulo 2 thì đa thức đã cho trở thành (x2 +x + 1)2

n. Chú ý rằng đa thức x2 + x + 1 là bất khả qui modulo 2.

Giả sử P(x) khả qui, tức tồn tại hai đa thức f , h đơn khởi với f , g ∈ Z[x], deg f , g ≥ 1sao cho P(x) = f (x).g(x). Khi đó f = (x2 + x + 1)k, g = (x2 + x + 1)2

n−k và

f (x) = (x2 + x + 1)k + 2 f0(x), g(x) = (x2 + x + 1)2n−k + 2g0(x),

với f0, g0 ∈ Z[x]. Gọi j là một căn bậc 3 khác 1 của đơn vị. Thay j vào đẳng thứcP(x) = f (x).h(x) ta được

P( j) = g( j).h( j)⇔ 2= 4 f0( j)g0( j).

Từ đó suy ra f0( j).g0( j) =12

. Do f0( j)g0( j) luôn viết được dưới dạng a j + b; a, b ∈ Z

và đẳng thức này không thể xảy ra. Ta có điều phải chứng minh.

Một số bài toán tương tự như sau

1. Với n≥ 1 là số tự nhiên, chứng minh các đa thức sau là bất khả qui trên Z

(a) P(x) = (x3 + x)2n − 3

(b) P(x) = (x2 + ax)2n+ 1 với a ∈ Z

Tập san Toán học STAR EDUCATION

Page 35: Möclöc - geosiro.com · Sßdöngph÷ìngph¡p˜⁄isŁtrong chøngminhh…nhhåc Nguy„n Tăng Vũ (Giáo viên PTNK TP Hç Chí Minh) Bài toán hình håc phflng thưíng xoay

VƯƠNG TRUNG DŨNG

2. Cho p là một số nguyên tố có dạng 4k+3. Chứng minh rằng với mọi số nguyêndương n đa thức P(x) = (x2 + 1)n + p bất khả qui trên Z[x].

3. Cho p là một số nguyên tố và a là một số nguyên không chia hết cho p. Chứngminh đa thức P(x) = x p − x + a bất khả qui trên Z[x].

Bài toán 4. (Japan 99) Chứng minh rằng đa thức f (x) = (x2 + 12)(x2 + 22)...(x2 +n2) + 1 là bất khả qui trên Z

Lời giải. Giả sử n ≥ 2 vì trường hợp n = 1 là tầm thường và giả sử f (x) = g(x).h(x)với f , g ∈ Z[x] và 1≤ deg f , g ≤ n− 1. Khi đó

1= f (±ki) = g(±ki)h(±ki).

Vì f , g ∈ Z[x] nên g(±ki), h(±ki) có dạng a+ bi . Từ đó suy ra

1= g(ki)h(ki) = 1.1= (−1).(−1) = i.(−i) = (−i).i

Như vậy trong tất cả 4 trường hợp ta đều có g(ki) = h(ki) = h(−ki), với k = 1, 2, ..., n.Như vậy đa thức P(x) = g(x)−h(−x) có 2n nghiệm phân biệt nhưng có bậc nhỏ hơn2n nên là đa thức 0 và do đó g(x) = h(−x). Suy ra deg(g) = deg(h) = n.

Vì f đơn khởi nên g, h cũng đơn khởi. Khi đó g2 − h2 có bậc không quá 2n− 1 nhưnglại có ít nhất 2n nghiệm ki, với i ∈ −n,−n−1, ...,−1,1, ..., n nên g2 = h2. Nhưng dễthấy g = −h không xảy ra do đó g ≡ h. Khi đó

f (x) = g(x)2⇒ f (0) = (g(0))2 = (n!)2 + 1,

vô lí. Bài toán được chứng minh xong.

Ta có bài toán tổng quát hơn là: Cho p là một số nguyên tố. Chứng minh rằng với mỗisố tự nhiên n đa thức

P(x) = (x p + 12)(x p + 22)...(x p + n2) + 1

bất khả qui trên Z[x].

Bài toán 5. Cho m, n, a là các số nguyên dương và số nguyên tố p thỏa mãn p < a−1.Chứng minh rằng đa thức P(x) = xm(x − a)n + p bất khả qui trên Z.

Lời giải. Giả sử P(x) khả qui và P(x) = G(x).H(x), với G, H ∈ Z[x]. Vì P(0) = p nên|G(0)| = 1 hoặc |H(0)| = 1. Không mất tổng quát ta giả sử G(x) = x k + ak−1 x k−1 +...+ a0 và |G(0)|= 1, m+ n− 1≥ k ≥ 1.

Gọi x1, ..., xk là nghiệm của G(x). Ta viết G(x) dưới dạng G(x) = (x − x1)...(x − xk),dẫn đến |x1...xk| = 1 và P(x i) = 0⇔ xm

i (x i − a)n = −p. Cho i = 1, k và nhân các vếcủa đẳng thức lại ta được

|G(x)|n = pk nên |G(a)|n = pk.

Mặt khác P(a) = G(a).H(a) = p nên ta suy ra |G(a)|= p. Do đó |G(a)−G(0)|= p±1chia hết cho a. Vì thế nên p−1≥ a hoặc p+1≥ a (mâu thuẫn với giả thiết p < a−1).Vậy P(x) là bất khả qui.

Tập san Toán học STAR EDUCATION

Page 36: Möclöc - geosiro.com · Sßdöngph÷ìngph¡p˜⁄isŁtrong chøngminhh…nhhåc Nguy„n Tăng Vũ (Giáo viên PTNK TP Hç Chí Minh) Bài toán hình håc phflng thưíng xoay

36 VƯƠNG TRUNG DŨNG

Bài toán 6. (Rumani 1999) Cho số nguyên a và số nguyên dương n và p là một sốnguyên tố thoả p > |a|+1. Chứng minh rằng đa thức P(x) = xn+ ax + p bất khả quitrên Z[x].

Lời giải. Giả sử P(x) = g(x).h(x), với g, h ∈ Z[x] và 1 ≤ deg f , deg g ≤ n − 1. VìP(0) = p = g(0).h(0) nên không mất tổng quát giả sử g(0) = ±1, h(0) = ±p. Khi đóg(x) = ±xm + T (x)± 1, T ∈ Z[x], degT ≤ m− 1.

Gọi z1, z2, . . . , zm là nghiệm của g(x). Theo định lí Viet 1 = |g(0)| = |z1z2...zm| nên|zi| ≤ 1, i = 1, 2, ..., m. Lại có 0= f (zi) = zn

i + azi + p nên

p = −zni + azi ≤ |zi|n + |a|.|zi| ≤ 1+ |a|,

vô lí. Vậy ta có điều phải chứng minh.

Bài toán 7. Cho p, q là hai số nguyên tố lẻ sao cho q không là ước của p − 1 và gọia1, a2, ..., an là các số nguyên phân biệt sao cho q|(ai − a j) với mọi cặp (i, j). Chứngminh rằng

P(x) = (x − a1)(x − a2)...(x − an)− p

là bất khả qui trên Z[x] với mọi n≥ 2.

Lời giải. Giả sử f khả qui trên Z[x]. Khi đó tồn tại Q(x), R(x) ∈ Z[x] sao choQ(x)R(x) = P(x) và 1≤ degQ(x),≤ degR(x). Nói riêng degQ(x)≤

n2

.

Ta có Q(ai) = R(ai) = −p, với 1 ≤ i ≤ n từ đó suy ra Q(ai), R(ai) ∈ −1, 1,−p, p với

mọi 1 ≤ i ≤ n. Với mọi hằng số c đa thức Q(x) − c nhận nhiều nhất degQ(x) ≤n2

nghiệm. Do đó Q(ai) nhận ít nhất hai giá trị phân biệt (và ít nhất 3 giá trị phân biệt

nếu degQ(x)<n2

).

Vì q|(ai− a j) nên q|(Q(ai)−Q(a j). Để ý rằng q là số nguyên tố lẻ nên Q(ai) không thểnhận hai giá trị 1 và −1 (vì nếu ngược lại thì q|1− (−1) = 2). Tương tự Q cũng khôngthể nhận hai giá trị là p− p vì khi đó R(ai) nhận hai giá trị là 1,−1.

Từ giả thiết q không là ước của p − 1 suy ra Q(ai) không thể nhận hai giá trị 1 và phoặc −1 và −p. Do đó Q(ai) chỉ có thể nhận được nhiều nhất hai giá trị là 1 và −phoặc −1 và p. Giả sử trường hợp đầu tiên xảy ra.

Vì Q(ai) chỉ nhận hai giá trị nên degQ(ai) =n2

và Q nhận mỗi giá trị 1 và −p đúngn2

lần. Chia tập (ai)ni=1 thành hai tập (bi)ni=1 và (ci)ni=1 sao cho Q(bi) = 1 và Q(ci) = −p.Khi đó

Q(x) = (x − b1)(x − b2)...(x − b n2) + 1= (x − c1)(x − c2)...(x − c n

2)− p.

Vì ta cũng có degR(x) =n2

nên R(ai) = −p khi Q(ai) = 1. Do đó

R(x) = (x − b1)(x − b2)...(x − b n2)− p = (x − c1)(x − c2)...(x − c n

2) + 1.

Nhưng khi đó để ý rằng trong đẳng thức thứ nhất cho ta Q(x)−R(x) = 1+p còn đẳngthức thứ 2 cho ta Q(x)−R(x) = −p− 1 điều đó dẫn đến p = −1. Vô lí. Bài toán đượcchứng minh xong.

Tập san Toán học STAR EDUCATION

Page 37: Möclöc - geosiro.com · Sßdöngph÷ìngph¡p˜⁄isŁtrong chøngminhh…nhhåc Nguy„n Tăng Vũ (Giáo viên PTNK TP Hç Chí Minh) Bài toán hình håc phflng thưíng xoay

VƯƠNG TRUNG DŨNG

Bài toán 8. Tìm tất cả các cặp số nguyên dương (m, n) sao cho đa thức

P(x , y) = (x + y)2(mx y + n) + 1

khả qui trên Z[x , y]. Khi đó hãy phân tích f thành các nhân tử bất khả qui.

Lời giải. Đặt S = x + y ta viết lại f dưới dạng

f (x , S) = S2(mx(S − x) + n) + 1= −mS2 x2 +mS3 x + (nS2 + 1).

Ta xem f là một tam thức bậc 2 theo biến x . Khi đó f khả qui khi và chỉ khi f phântích được thành tích của hai đa thức bậc nhất. Khi đó biệt thức

∆= m2S6 + 4mS2(nS2 + 1) = mS2(mS4 + 4nS2 + 4)

là một bình phương.

Dễ thấy điều này xảy ra khi và chỉ khi m= n2, lúc này

∆= (nS(nS2 + 2))2

và f có hai nghiệm là x =−1nS

hoặc x =nS2 + 1

nS.

Khi đó

f (x) = (nSx + 1)(−nSx + nS2 + 1) = (nx2 + nx y + 1)(ny2 + nx y + 1).

7. Bài tập tự luyện

1. Với n ≥ 2 là một số nguyên và r = np2. Chứng minh rằng không tồn tại các sốhữu tỷ a0, a1, ..., an−1 không đồng thời bằng 0 sao cho

a0 + a1r + a2r2 + ...+ an−1rn−1 = 0

2. Tìm số nguyên dương n nhỏ nhất sao cho đa thức P(x) = xn−4+4n có thể phântích được thành tích của 4 đa thức hệ số nguyên và không là đa thức hằng.

3. Cho P(x),Q(x) là hai đa thức đơn khởi, bất khả quy trên trường số hữu tỷ. Giảsử P,Q có hai nghiệm tương ứng là α,β sao cho α+β là số hữu tỷ. Chứng minhP2(x)−Q2(x) có nghiệm hữu tỷ.

4. Chứng minh đa thức P(x) = (1+ x + x2 + ...+ xn)2 − xn khả qui trên Z[x].

5. Chứng minh rằng đa thức P(x) = xn + 4 khả qui trên Z khi và chỉ khi n là bộicủa 4.

6. (IMO Longlist 1989) Cho n ≥ 4 và các số nguyên phân biệt a1, a2, ..., an. Chứngminh đa thức

P(x) = (x − a1)(x − a2)...(x − an)− 2

bất khả qui trên Q[x].

Tập san Toán học STAR EDUCATION

Page 38: Möclöc - geosiro.com · Sßdöngph÷ìngph¡p˜⁄isŁtrong chøngminhh…nhhåc Nguy„n Tăng Vũ (Giáo viên PTNK TP Hç Chí Minh) Bài toán hình håc phflng thưíng xoay

38 VƯƠNG TRUNG DŨNG

7. (VMO 2014) Cho n là số nguyên dương. Chứng minh rằng đa thức P(x) =(x2−7x +6)n+13 không thể biểu diễn được thành tích của n+1 đa thức kháchằng với hệ số nguyên.

8. Chứng minh rằng đa thức xn − x − 1 bất khả qui trên Q[x], với mọi n≥ 2.

9. Cho n> m> 1 là hai số nguyên lẻ. Chứng minh đa thức P(x) = xn+ xm+ x +1bất khả qui trên Z[x].

10. Cho p là số nguyên tố. Chứng minh rằng đa thức

P(x) = x p−1 + 2x p−2 + ...+ (p− 1)x + p

bất khả qui trên Z.

11. Cho đa thức P(x) = an xn + an−1 xn−1 + ... + a1 x + a0 ∈ Z[x], (an 6= 0, n ≥ 2).Chứng minh rằng tồn tại vô số số nguyên tố k sao cho đa thức P(x)+ k bất khảqui.

12. Tìm tất cả các số nguyên n sao cho đa thức P(x) = x5 − nx − n− 2 là khả quitrên Z[x].

13. Cho p là một số nguyên tố và n là một số nguyên nhỏ hơn 4. Chứng minh rằngnếu a là một số nguyên không chia hết cho p thì đa thức P(x) = axn − px2 +px + p2 bất khả qui trên Z[x].

14. Cho p là số nguyên tố. Chứng minh rằng đa thức P(x) = x p + (p − 1)! bất khảqui trên Z[x].

15. Tồn tại hay không đa thức f ∈ Q[x] sao cho f (1) 6= −1 và xn f (x) + 1 là khảqui với mọi n ∈ N.

16. Cho a là một số nguyên dương và p ≥ 2 là một số nguyên tố thỏa mãn (a, p) =1. Chứng minh rằng đa thức P(x) = x p − mx + a bất khả qui trên Z[x] vớim≡ 1 (mod p).

17. Cho p là một số nguyên tố lẻ. Chứng minh đa thức P(x) =p−2∑

i=0(p − 1− i)x i bất

khả qui trên Q[x].

18. (Rumani TST 2003) Cho P(x) ∈ Z[x] là một đa thức monic bất khả qui trênZ[x] sao cho P(0) không là số chính phương. Chứng minh rằng Q(x) = P(x2)cũng bất khả qui trên Z[x].

19. (China TST 2006) Cho số nguyên n ≥ 2. Chứng minh rằng tồn tại đa thứcP(x) = xn + an−1 xn−1 + ...+ a1 x + a0 thỏa mãn

(a) a0, a1, ..., an−1 khác 0.

(b) P(x) bất khả qui.

(c) Với mọi số nguyên x thì |P(x)| không là số nguyên tố.

Tập san Toán học STAR EDUCATION

Page 39: Möclöc - geosiro.com · Sßdöngph÷ìngph¡p˜⁄isŁtrong chøngminhh…nhhåc Nguy„n Tăng Vũ (Giáo viên PTNK TP Hç Chí Minh) Bài toán hình håc phflng thưíng xoay

VƯƠNG TRUNG DŨNG

20. Biết f ∈ Z[x] là một đa thức bất khả qui có bậc lẻ và lớn hơn 3. Giả sử rằngcác nghiệm của P đều có modun lớn hơn 1 và f (0) không có ước chính phương.Chứng minh rằng đa thức g(x) = f (x3) cũng là đa thức bất khả qui.

21. Cho f ∈ Z[x] là một đa thức monic với bậc lớn hơn 1. Giả sử f (xn) bất khả quitrên Z[x] với mọi n≥ 2. Hỏi f có bất khả qui trên Z[x] hay không?

22. Cho 1 6= f ∈ Z[x] sao cho có vô hạn số nguyên a thỏa f (x2 + ax) bất khả quitrên Q[x]. Hỏi f có bất khả qui trên Q[x] hay không?

23. Cho f (x) 6= ±x là một đa thức bất khả qui trên Z[x]. Hỏi f (x .y) có bất khả quitrên Z[x , y] hay không?

Tập san Toán học STAR EDUCATION

Page 40: Möclöc - geosiro.com · Sßdöngph÷ìngph¡p˜⁄isŁtrong chøngminhh…nhhåc Nguy„n Tăng Vũ (Giáo viên PTNK TP Hç Chí Minh) Bài toán hình håc phflng thưíng xoay

Tài liệu tham khảo

[1] Nguyễn Tiến Quang, NXB Giáo dục Đại số đại cương

[2] Đoàn Duy Cường, 2015 Bài giảng bồi dưỡng giáo viên chuyên toán năm

[3] Nguyễn Chu Gia Vượng,2015, Đa thức bất khả qui

[4] Exploration-Creativity 2012, Irreducible polynomials

[5] Yufei Zhao, Integer polynomial

[6] Dusan Djukic, Polynomials in one variable .

[7] Gabriel D.Carroll, Polynomials .

[8] Victor V.Prasolov, Polynomials

[9] Titu Andresscu, Gabriel Dospinescu, Problems from the book

[10] U298, Mathematical Reflections

[11] https://artofproblemsolving.com/community/c89

40

Page 41: Möclöc - geosiro.com · Sßdöngph÷ìngph¡p˜⁄isŁtrong chøngminhh…nhhåc Nguy„n Tăng Vũ (Giáo viên PTNK TP Hç Chí Minh) Bài toán hình håc phflng thưíng xoay

Các bài toán tổ hợp trên dãy số

Lê Phúc Lữ(Lớp Cao học Khoa học tự nhiên TP.HCM)

Trong bài viết nhỏ này, chúng ta sẽ cùng xét khía cạnh tổ hợp của dãy số nguyên; khicần đếm số lượng dãy thỏa mãn một điều kiện cho trước nào đó. Các phương phápthường gặp: truy hồi, xuống thang, cực hạn, phản chứng, ...

1. Các bài toán chọn lọc

Bài toán 1. Tìm tất cả các bộ số nguyên dương x1, x2, x3, . . . , x2017 sao cho có thể đặtchúng lên vòng tròn theo thứ tự đó mà 6 số liên tiếp bất kỳ đều có thể chia thành hainhóm 3 có tổng bằng nhau.

Lời giải. Dùng phương pháp xuống thang. Ta có x i+ x i+1+ x i+2+ x i+3+ x i+4+ x i+5 ≡ 0(mod 2) với mọi i = 1,2, 3, . . . , 2017 nên x i ≡ x i+6 với mọi i. Vì (6, 2017) = 1 nên suyra tất cả các số có cùng tính chẵn lẻ. Ta xét phép biến đổi dãy số sau:

1. Nếu tất cả các số cùng chẵn thì thay bằng yi =x i2 .

2. Nếu tất cả các số cùng lẻ thì thay bằng yi =x i+1

2 .

Dễ thấy dãy mới cũng thỏa và tổng S =2017∑

i=1ai sẽ giảm ngặt nếu có một số nào đó

trong dãy khác 1; suy ra quá trình biến đổi sẽ dừng lại khi tất cả đều là 1. Vì ta thuđược một dãy toàn là 1 nên dãy ban đầu có tất cả các số hạng bằng nhau.

Nhận xét. Bài toán trên có thể thay việc chia 2 nhóm thành 3, 4,5, . . . nhóm và vẫngiải được bằng cách tương tự. Ta xét các bài tương tự sau:

Bài toán 2. (APMO 2017) Bộ năm số nguyên là tốt nếu có thể đặt chúng là a, b, c, d, eđể a− b+ c− d + e = 29. Tìm tất cả các bộ 2017 số sao cho 5 số liên tiếp bất kỳ trongchúng đều tốt.

Ở bài toán này, điểm khó là không biết các số đã cho có dương hay không; vì thể, đạilượng tổng ở trên không xét tiếp tục được. Tuy nhiên, cách áp dụng vẫn tương tự nhưsau:

• Trừ tất cả các số của bộ cho 29, ta thu được điều kiện tốt trở thành a− b+ c −d + e = 0.

41

Page 42: Möclöc - geosiro.com · Sßdöngph÷ìngph¡p˜⁄isŁtrong chøngminhh…nhhåc Nguy„n Tăng Vũ (Giáo viên PTNK TP Hç Chí Minh) Bài toán hình håc phflng thưíng xoay

42 LÊ PHÚC LỮ

• Tất cả các số đã cho cùng tính chẵn lẻ, và chính xác là cùng chẵn.

• Xét đại lượng S =2017∑

i=1

ai2

thì thông qua phép chia 2, tổng này giảm ngặt. Từ đó

suy ra tất cả các số này phải là 0 và tất cả ban đầu phải là 29.

Bài toán 3. (VMO 2014) Tìm tất cả các bộ số 2014 số hữu tỷ không âm sao cho nếubỏ đi bất kỳ số nào trong chúng thì các số còn lại có thể được chia thành 3 nhóm rờinhau, mỗi nhóm có 671 số sao cho tích các số trong mỗi nhóm là bằng nhau.

Bài này khó hơn vì: số hữu tỷ chứ không nguyên, tích chứ không phải tổng, ... Ta lầnlượt giải quyết điều đó như sau:

• Quy đồng mẫu để đưa về số nguyên.

• Xét số mũ của 1 ước nguyên tố để đưa về tổng.

• Chú ý thêm trường hợp số 0 (nếu có 1 số thì phải có ít nhất 4 số).

Bài toán 4. Cho dãy số nguyên dương (an) thỏa mãn:

i) Gồm các số phân biệt nhau.

ii) Với mọi n thì an ≥ n.

iii) a1 = 5, a2 = 4, a3 = 3.

1. Chứng minh rằng tồn tại n> 2017 sao cho an 6= n+ 1?

2. Giả sử an = n+ 2 với mọi n> 2017, hỏi có tất cả bao nhiêu dãy số như thế?

Lời giải. 1) Bài toán có thể giải quyết dễ dàng bằng phản chứng và Dirichlet. Thậtvậy, nếu an = n+ 1 với mọi n > 2017 thì các số hạng a4 → a2017 sẽ nhận các giá trịtrong tập hợp 6→ 2018. Khi đó, sẽ có hai số hạng bằng nhau, không thỏa.

2) Nếu đã có an = n+ 2 với mọi n > 2017 thì các số hạng a4→ a2017 sẽ nhận các giátrị trong tập hợp 6→ 2019. Nhận xét:

• a2017 ∈ 2017,2018, 2019 nên có 3 cách chọn.

• a2016 ∈ 2016,2017, 2018,2019 nhưng vì a2017 đã lấy một số nên cũng còn 3cách chọn.

• Tương tự, đến a6 vẫn có 3 cách chọn. Còn lại a5 có 2 cách chọn và a4 có 1 cáchchọn.

Theo nguyên lý nhân, ta có 2 · 32012 dãy thỏa mãn.

Tập san Toán học STAR EDUCATION

Page 43: Möclöc - geosiro.com · Sßdöngph÷ìngph¡p˜⁄isŁtrong chøngminhh…nhhåc Nguy„n Tăng Vũ (Giáo viên PTNK TP Hç Chí Minh) Bài toán hình håc phflng thưíng xoay

LÊ PHÚC LỮ

Bài toán 5. Xét lục giác ABC DEF có độ dài cạnh là 1 được điền các số như hình vẽMột con ếch xuất phát từ A và nhảy đến các đỉnh sao cho mỗi bước nhảy đều có độdài nguyên. Hành trình của ếch là dãy các tên đỉnh mà ếch đã nhảy qua; và hai hànhtrình được coi là khác nhau nếu ở một lần thứ k nào đó, đỉnh mà ếch nhảy đến ở haihành trình là khác nhau.

Gọi m là số hành trình ếch nhảy sao cho tổng các số mà nó nhảy qua là 2017. Chứngminh rằng m không phải là số chính phương.

Lời giải. Ta thấy AC E và BDF là hai tam giác đều có cạnh làp

3 nên mỗi lần, ếch sẽnhảy từ tam giác đều này đến tam giác đều kia. Chia nhóm:

• I = (A, C , E) tương ứng với các số (0, 0,1).

• I I = (B, D, F) tương ứng với (1,1, 2).

Ta thấy x + y|x ∈ I , y ∈ I I = 1, 1,1, 1,2, 2,2, 2,3 chứng tỏ tổng các số trên haibước nhảy liên tiếp của ếch sẽ nhận giá trị là 4 số 1, 4 số 2 và 1 số 3. Nếu gọi sn là sốhành trình của ếch có tổng là n thông qua chẵn bước thì

sn = 4sn−1 + 4sn−2 + sn−3.

Một cách tương tự, gọi tn là số hành trình của ếch có tổng là n thông qua lẻ bước thìcông thức truy hồi vẫn thế (chỉ khác ở các số hạng đầu). Vì vậy nên nếu gọi un = sn+ tn

là số hành trình của ếch có tổng là n thì

un = 4un−1 + 4un−2 + un−3 với n≥ 3.

Ta có u0 = 1, u1 = 6, u2 = 28 và từ công thức truy hồi thì m = u2017 ≡ u1 ≡ 2 (mod 4)nên m không thể là số chính phương, ta có đpcm.

Nhận xét. Bài toán có thể giải bằng cách gọi 6 dãy truy hồi an, bn, cn, dn, en, fn chỉ sốhành trình của ếch có tổng là n và kết thúc tại A, B, C , D, E, F . Tuy nhiên, cách tiếp cậnđó khá phức tạp, đòi hỏi phải khai thác nhiều các liên hệ giữa các đường đi. Một bàitoán tương tự:

Tập san Toán học STAR EDUCATION

Page 44: Möclöc - geosiro.com · Sßdöngph÷ìngph¡p˜⁄isŁtrong chøngminhh…nhhåc Nguy„n Tăng Vũ (Giáo viên PTNK TP Hç Chí Minh) Bài toán hình håc phflng thưíng xoay

44 LÊ PHÚC LỮ

Bài toán 6. (Ả Rập TST 2017) Người ta đặt các số 1, 2,3, 4 trên vòng tròn theo thứtự đó. Một con kiến xuất phát từ số 1 và ở mỗi bước, nó sẽ bò qua số bên cạnh. Hỏicon kiến có bao nhiêu cách bò sao cho tổng tất cả các số mà nó bò qua (kể cả số banđầu) bằng 21?

Tương tự bài trên, ta cũng tìm được hệ thức truy hồi là sn = sn−3 + 2sn−5 + sn−7. Từ đótính được s21 = 167.

Bài toán 7. Đếm số dãy số nguyên dương (a1, a2, . . . , a12) thỏa mãn các điều kiện sau:

1. 1≤ a1 ≤ a2 ≤ . . .≤ a12 ≤ 2017

2. ai ≡ i2(mod12).

Lời giải. Theo giả thiết, ta có

a1 ≡ a5 ≡ a7 ≡ a11 ≡ 1 (mod12)a2 ≡ a4 ≡ a8 ≡ a10 ≡ 4 (mod12)a3 ≡ a9 ≡ 9 (mod12)a6 ≡ a12 ≡ 0 (mod12)

Đặt ai = 12bi + ri với i = 1, 2,3, . . . , 12 và ri là số dư tương ứng đã chỉ ra ở trên. Dotính không giảm của dãy nên ta phải có

0≤ b1 ≤ b2 ≤ b3 < b4 < b5 < b6 ≤ b7 ≤ b8 ≤ b9 < b10 < b11 < b12 ≤ 168.

Từ đó suy ra

0≤ b1 < b2 + 1< b3 + 2< b4 + 2< b5 + 2< b6 + 2≤ b7 + 3≤ b8 + 4≤ b9 + 5

< b10 + 5< b11 + 5< b12 + 5≤ 173

Do các số liệt kê ở trên đều phân biệt và thuộc [0;173] nên số cách chọn một bộ nhưthế là C12

174. Đó cũng chính là số dãy cần tìm.

Nhận xét. Điều kiện thứ hai có thể thay bằng một hàm số tùy ý theo i chứ không nhấtthiết phải là i2, cách giải vẫn tương tự như trên.

Bài toán 8. Hỏi có bao nhiêu hoán vị a1, a2, ..., a2017 của 2017 số nguyên dương đầutiên thỏa mãn đồng thời các điều kiện sau:

i) ai+1 − ai ≤ 1 với mọi i = 1, 2,3, . . . , 2016.

ii) Có đúng một chỉ số i với 1≤ i ≤ 2017 sao cho ai = i?

Lời giải. Trước hết, ta sẽ chứng minh nhận xét rằng số hoán vị của n số nguyên dươngđầu tiên thỏa mãn điều kiện i), gọi là hoán vị đẹp, sẽ là 2n−1. Thật vậy,

• Đầu tiên, ta đặt số 1 vào hoán vị.

Tập san Toán học STAR EDUCATION

Page 45: Möclöc - geosiro.com · Sßdöngph÷ìngph¡p˜⁄isŁtrong chøngminhh…nhhåc Nguy„n Tăng Vũ (Giáo viên PTNK TP Hç Chí Minh) Bài toán hình håc phflng thưíng xoay

LÊ PHÚC LỮ

• Số 2 có thể xếp trước hoặc sau số 1, có 2 cách.

• Số 3 có thể xếp vào đầu dãy hoặc ngay sau số 2 đã xếp trước đó, có 2 cách.

• Số 4 có thể xếp vào đầu dãy hoặc ngay sau số 3 đã xếp trước đó, cũng có 2 cách.Cứ như thế cho đến n.

Do đó, có tất cả 2n−1 cách xếp, tương ứng vời 2n−1 hoán vị. Tiếp theo, giả sử ta cóai = i. Khi đó ai+1 ≤ ai + 1 = i + 1, nhưng không thể có ai+1 = i + 1 (do chỉ có 1 chỉsố thỏa mãn ii) nên ai+1 ≤ i, mà ai = i nên ai+1 ≤ i − 1. Tiếp theo, ai+2 ≤ ai+1 + 1≤ inên ai+2 ≤ i − 1.

Do đó, các số từ ai+1 đến a2017 nhận giá trị không vượt quá i − 1.

Lập luận tương tự, các số từ a1 đến ai−1 phải nhận giá trị không nhỏ hơn i + 1.

Do đó, hai đoạn hoán vị phía trước và phía sau ai phải có độ dài bằng nhau, tức làa1009 = 1009 là số ở giữa. Rõ ràng các hoán vị phía trước và phía sau 1009 đều phảilà hoán vị đẹp và được sắp xếp độc lập với nhau.

Vậy số hoán vị cần tìm là

210072= 22014.

Nhận xét. Nếu đề đổi số 2017 thành 2018 thì sẽ tồn tại hai chỉ số i như trên và chúngsẽ cách đều hai đầu 1 và 2018. Khi đó, đoạn ở giữa cũng sẽ cố định, tức là có i < jđể ak = k với mọi k = i, i + 1, . . . , j và i + j = 2019. Phần trước i và phần sau j sẽ đổichỗ cho nhau với số cách xếp là (2i−1)2.

Bài toán 9. Cho dãy các số nguyên dương (un) thỏa mãn điều kiện 0 ≤ um+n − um −un ≤ 1 với mọi m, n ∈ Z+. Chứng minh rằng tồn tại a ∈ R+ sao cho−1≤ un−[an]≤ 1với mọi n= 1,2, 3, . . . , 2017.

Lời giải. Ta đưa điều cần chứng minh về

un

n< a <

un + 1n

.

Đến đây, gọi

m=min

un + 1n

n= 1,2, 3, . . . , 2017

vàM =max

n un

n

n= 1, 2,3, . . . , 2017.o

Cần chỉ ra m > M rồi chọn số a nằm giữa (m, M) là xong. Gọi p, q lần lượt là cácchỉ số nhỏ nhất để có dấu bằng xảy ra ở các đánh giá trên. Khi đó up + 1 = pm vàuq = Mq. Ngoài ra, uk + 1> km,∀k < p và uk < kq,∀k < q.

• Nếu p = q thì hiển nhiên đúng.

• Nếu p > q, ta đặt p = q+ k thì k < p nên uk + 1> km, vì up ≥ uq + uk (theo giảthiết) nên pm− 1> Mq+ km− 1⇔ m> M .

• Nếu p < q thì cũng chứng minh tương tự với chú ý rằng uq ≤ up + uk + 1.

Tập san Toán học STAR EDUCATION

Page 46: Möclöc - geosiro.com · Sßdöngph÷ìngph¡p˜⁄isŁtrong chøngminhh…nhhåc Nguy„n Tăng Vũ (Giáo viên PTNK TP Hç Chí Minh) Bài toán hình håc phflng thưíng xoay

46 LÊ PHÚC LỮ

Nhận xét. Nếu đề bài đổi giả thiết thành 0≤ um+n− um− un ≤ 2, ta sẽ cần đến hai sốa, b sao mới thỏa mãn được kết luận (vì khoảng chênh lệch của các số hạng rộng hơnmột tí), cụ thể là tồn tại a, b > 0 để −1 ≤ un − [an]− [bn] ≤ 1. Ở bài toán trên, tacòn chứng minh được một kết quả mạnh hơn là tồn tại a để un = [an] với mọi n. Mộtbài toán tương tự trong đề trường Đông miền Trung:

Bài toán 10. Cho hàm số f : R→ R thỏa mãn | f (x + y)− f (x)− f (y)| ≤ 1,∀x , y ∈R. Chứng minh rằng tồn tại hàm cộng tính g : R → R thỏa mãn | f (x)− g(x)| ≤1,∀x .

Đây là có thể nói là một phiên bản trên R của bài toán trên (thay vì xét trên N).

Tiếp theo, ta xét lớp các bài toán sử dụng một định lý thú vị trong dãy số, số học.Trước hết, ta xét định lý Beatty với nội dung như sau:

Cho hai số vô tỷ dương α,β . Xét hai dãy số:

• [α], [2α], [3α], . . . tạo thành dãy A.

• [β], [2β], [3β], . . . tạo thành dãy B.

Khi đó 1α +

1β = 1 khi và chỉ khi A, B là phân hoạch của Z+.

Định lý này có thể chứng minh bằng cách sử dụng các BĐT về phần nguyên. Dưới đâylà cách chứng minh cho chiều đảo:

Với mỗi số nguyên dương k, gọi m, n là các số nguyên dương thỏa mãn

[mα]≤ k < [(m+ 1)α] và [nβ]≤ k < [(n+ 1)β].

Đặt A= [iα], 1≤ i ≤ m và B = [ jβ], 1≤ j ≤ n thì |A|= m, |B|= n và A, B là phânhoạch của tập hợp 1, 2,3, . . . , k theo định nghĩa của đề bài.

Do đó m+ n = k. Theo bất đẳng thức phần nguyên thì mα− 1 < k < (m+ 1)α nênm

k+1 <1α <

m+1k . Tương tự n

k+1 <1β <

n+1k . Suy ra

m+ nk+ 1

<1α+

1β<

m+ n+ 2k

hayk

k+ 1<

1α+

1β<

k+ 2k

.

Cho k→ +∞, ta thu được 1α +

1β = 1.

Bài toán 11. Hai dung dịch A, B có đặc điểm: số đo thể tích của 1 kg A bằng số đokhối lượng của 1 lít B. Ngoài ra, p lít A nặng bằng q lít B với p, q nguyên tố khácnhau. Mỗi dung dịch được chia cho vào các bình nhỏ giống nhau, cùng chứa 1 lít vàvỏ nặng 1 kg. Chứng minh rằng có đúng một cách ghép các bình cùng loại (A hoặc B)lại với nhau mà khối lượng của chúng thuộc khoảng (2017; 2018).

Lời giải. Gọi x , y lần lượt là khối lượng riêng của các dung dịch thì 1x = 1 · y, px = q y

nên x =q

qp , y =

q

pq . Khối lượng mỗi bình là α = 1 +

q

qp ,β = 1 +

q

pq . Dễ thấy

1α +

1β = 1, thỏa mãn định lý Beatty. Suy ra hai dãy [mα], [nβ] là phân hoạch của số

nguyên dương nên ta có đpcm.

Tập san Toán học STAR EDUCATION

Page 47: Möclöc - geosiro.com · Sßdöngph÷ìngph¡p˜⁄isŁtrong chøngminhh…nhhåc Nguy„n Tăng Vũ (Giáo viên PTNK TP Hç Chí Minh) Bài toán hình håc phflng thưíng xoay

LÊ PHÚC LỮ

Bài toán 12. (APMO 2006) Với mỗi số nguyên dương n, gọi an, bn lần lượt là số cáchviết 10n trong hệ nhị phân, ngũ phân. Chứng minh rằng (an), (bn) là phân hoạch củaZ+\1.

Lời giải. Để giải bài này, chú ý rằng: số chữ số của M trong hệ p phân là [logpM]+1.Ngoài ra, α = log210,β = log510 thỏa mãn điều kiện của định lý Beatty. Từ đó, ta cómột nhận xét thú vị rằng: tổng số chữ số của 2n và 5n trong hệ thập phân là n+1.

Bài toán 13. (VN TST 2000) Cho số nguyên dương k. Dãy số (un) xác định bởi: u1 = 1và un+1 là số nguyên dương nhỏ nhất không thuộc tập hợp

u1, u2, . . . , un, u1 + k, u2 + 2k, . . . , un + nk .

Chứng minh rằng tồn tại α vô tỷ dương sao cho un = [nα] với mọi n.

Lời giải. Để giải bài toán này, ta xét đa thức P(x) = x2 + (k − 2)x − k với k là sốnguyên dương đã cho thì P(x) có hai nghiệm phân biệt trái dấu. Hơn nữa, ∆P(x) =(k− 2)2 + 4k = k2 + 4, không thể là số chính phương với bất kì số k nguyên dươngnào nên hai nghiệm này đều là số vô tỉ. Ta thấy

P(1) = 1+ (k− 2)− k = −1< 0, P(2) = 4+ 2(k− 2)− k = k > 0

nên nghiệm dương của phương trình P(x) = 0 thuộc khoảng (1, 2). Gọi nghiệm đó làa. Đặt b = a + k thì a, b đều vô tỉ và ab = a(a + k) = a2 + ak = 2a + k = a + b nên1a +

1b = 1. Xét f (n) = [na], g(n) = [nb] = f (n) + kn với n là số nguyên dương. Ta sẽ

chứng minh rằng xn = f (n) bằng quy nạp. Thật vậy,

- Với n= 1, khẳng định hiển nhiên đúng vì 1< a < 2.

- Giả sử xn = f (n) với mọi n = 1, 2,3, ..., m. Ta sẽ chứng minh rằng xm+1 = f (xm+1).Ta có f (i) = x i, g(i) = f (i) + ik = x i + ik với mọi i = 1, 2,3, ..., m nên ta có tập hợp

H = x1, x2, ..., xm, x1 + k, x2 + 2k, ..., xm +mk= f (1), f (2), ..., f (m), g(1), g(2), ..., g(m)

Rõ ràng f (m+ 1) /∈ H và g(n) > f (n) với mọi n, f (n) là hàm số đồng biến trên N∗nên ta thấy rằng f (m+ 1) chính là số tự nhiên nhỏ nhất không thuộc H. Theo địnhnghĩa dãy số (xn) đã cho thì ta có xm+1 = f (m+ 1).

Do đó, khẳng định cũng đúng với m+ 1. Theo nguyên lí quy nạp, ta có đpcm. Vậy sốtự nhiên cần tìm chính là a là nghiệm dương của phương trình x2+ (k− 2)x − k = 0.

Nhận xét. Đây là một kết quả có từ 1959. Ta có thể phân tích cách tiếp cận như sau:

Xuất phát từ việc α=p

2,β =p

2+2 thỏa mãn điều kiện Beatty. Ta có hai dãy với côngthức an =

np

2

, bn = an + 2n là phân hoạch của Z+. Từ đó, để giấu dãy bn đi, ta chỉcần xét an+ 2n. Để ý a1 = 1, a2 = 2, a3 = 4, b1 = 3, b2 = 6, b3 = 10 nên a4 có thể địnhnghĩa là số nguyên dương nhỏ nhất không thuộc a1, a2, a3, a1 + 2, a2 + 4, a3 + 6. Đóchính là cơ sở để có bài toán trên.

Tập san Toán học STAR EDUCATION

Page 48: Möclöc - geosiro.com · Sßdöngph÷ìngph¡p˜⁄isŁtrong chøngminhh…nhhåc Nguy„n Tăng Vũ (Giáo viên PTNK TP Hç Chí Minh) Bài toán hình håc phflng thưíng xoay

48 LÊ PHÚC LỮ

Bài toán 14. (Dãy Wythoff) Cho chuỗi S1 = 1. Chuỗi Sn được tạo thành từ chuỗi Sn−1

bằng cách thay 1→ 01 và 0→ 1. Các chuỗi S1, S2, S3, . . . được ghép liên tiếp lại vớinhau thành một chuỗi vô hạn L. Gọi an là vị trí của số 1 thứ n trong chuỗi L. Chứngminh rằng tồn tại α vô tỷ dương sao cho an = [nα] ,∀n.

Ở đây, ta có nhận xét rằng số 0 thứ n được sinh ra bởi số 1 thứ n nên nếu gọi kn là sốcác số 0 đứng trước số 1 thứ n và bn là vị trí của số 0 thứ n, ta sẽ có an = n+ kn vàbn = 2n+ kn nên bn = an + n. Chú ý rằng (an), (bn) chính là phân hoạch của Z+ nêndễ dàng tìm được α là nghiệm của 1

α +1α+1 = 1 hay α chính là tỷ số vàng.

Bài toán 15. Cho n là số nguyên dương, hỏi có bao nhiêu dãy số a1, a2, . . . , a2n saocho

i) ai ∈ −1,1 với i = 1, 2,3, . . . , 2n.

ii)

2l∑

i=2k+1ai

≤ 2 với 0≤ k < l ≤ n?

Lời giải. Gọi S là tập hợp các dãy thỏa mãn đề bài và đặt |S| = sn. Gọi T là tập hợptất cả các tổng các ai lấy từ chỉ số lẻ bất kỳ đến 2n. Theo giả thiết thì T ⊂ ±2,±1,0,tuy nhiên, tất cả các tổng trong T đều có chẵn số hạng mà mỗi số hạng đều là ±1nên tất cả phải đều chẵn. Suy ra T ⊂ ±2, 0. Nếu trong T chứa cả 2 lẫn −2 thì giả sử

2n∑

k=2i+1ak = 2 và

2n∑

k=2 j+1ak = −2 với i < j , khi đó

4=2n∑

k=2i+1

ak −2n∑

k=2 j+1

ak =2 j∑

k=2i+1

ak,

mâu thuẫn. Ứng với (a1, a2, . . . , a2n) ∈ S, ta có phân loại sau:

• Tất cả các tổng trong T đều là 0, đặt số lượng dãy có tính chất này là an.

• Trong T có chứa số 2, đặt số lượng dãy có tính chất này là bn.

• Trong T có chứa số −2, đặt số lượng dãy có tính chất này là cn.

Từ đó, ta dễ dàng chứng minh được hệ thức truy hồi

an+1 = 2an

bn+1 = an + 2bn + cn

cn+1 = an + bn + 2cn

Chú ý rằng an = 2n và an + bn + cn = sn. Cộng hai công thức cuối lại, ta có

bn+1 + cn+1 = 2an + 3(bn + cn)⇔ sn+1 − 2n+1 = 2n+1 + 3(sn − 2n)

haysn+1 = 3sn + 2n⇔ sn+1 + 2n+1 = 3(sn + 2n).

Với n= 1, ta có 4 dãy là (1, 1), (−1,−1), (−1,1), (1,−1) nên s1 = 4. Từ đẳng thức trên,ta có sn + 2n = 3n−1(s1 + 21) = 2 · 3n nên sn = 2 · 3n − 2n.

Tập san Toán học STAR EDUCATION

Page 49: Möclöc - geosiro.com · Sßdöngph÷ìngph¡p˜⁄isŁtrong chøngminhh…nhhåc Nguy„n Tăng Vũ (Giáo viên PTNK TP Hç Chí Minh) Bài toán hình håc phflng thưíng xoay

LÊ PHÚC LỮ

Nhận xét. Bài toán thoạt nhìn có vẻ quen thuộc nhưng thật không đơn giản. Điềukiện đề cho là giá trị tuyệt đối của tất cả các tổng con từ vị trí lẻ đến vị trí chẵn bất kỳđều không vượt quá 2 buộc ta phải có đánh giá thích hợp mới có thể truy hồi được.

2. Bài tập áp dụng

1. (TP.HCM 2018) Hỏi có bao nhiêu hoán vị (a1, a2, . . . , a164) của 164 số nguyêndương đầu tiên sao cho ai 6= i và ai ≡ i (mod41) với mọi i = 1, 2, . . . , 164?

2. (Bài toán phát kẹo) Cô giáo có 10 loại kẹo (mỗi loại có nhiều viên) và cần phátcho 30 học sinh của lớp (một em nhận không quá 1 viên/loại), giả sử rằng cácem này có học lực đôi một khác nhau. Hỏi cô giáo có bao nhiêu cách phát kẹo,biết rằng nếu học sinh A giỏi hơn B thì B có kẹo gì là A có kẹo đó (tính cả trườnghợp không em nào nhận được kẹo)?

3. (Bài toán con nhện) Một con nhện có 8 cái chân, 8 cặp vớ - giày khác nhau (vớchỉ dùng chung với chiếc giày tương ứng). Con nhện có bao nhiêu thứ tự mangvớ và giày để sao cho trên cùng một chân, giày phải được mang vào sau vớ?

Tập san Toán học STAR EDUCATION

Page 50: Möclöc - geosiro.com · Sßdöngph÷ìngph¡p˜⁄isŁtrong chøngminhh…nhhåc Nguy„n Tăng Vũ (Giáo viên PTNK TP Hç Chí Minh) Bài toán hình håc phflng thưíng xoay

Đề thi tham khảo hướng tới VMO 2019

Kỳ thi chọn HSG cấp quốc gia (viết tắt là VMO) năm nay diễn ra vào các ngày 13 và14 tháng 01/2019. Trong bài viết này, Ban biên tập chúng tôi sẽ đưa ra giới thiệu mộtđề thi thử cùng đáp án và phân tích chi tiết.

1. Đề thi tham khảo

1.1. Đề thi ngày thứ nhất

Bài 1. Cho dãy số dương (un) xác định bởi u1 =13 , u2 = 3. Giả sử:

1.1+punun+1p

1+ un

=1+pun+1un+2p

1+ un+1

với n≥ 1. Tìm giới hạn của dãy (un).

2.1+punun+2p

1+ un

=1+pun+1un+2p

1+ un+1

với n≥ 1. Chứng minh rằng lim un = tan2 5π18 .

Bài 2. Cho tam giác ABC nhọn, không cân nội tiếp đường tròn (O) có trực tâm H và Mlà trung điểm của AH. Gọi (O1), (O2) là các đường tròn đi qua H và lần lượt tiếp xúc vớiBC ở B, C . Gọi X , Y lần lượt là tâm bàng tiếp góc H của hai tam giác HMO1, HMO2.

1. Chứng minh rằng X Y song song với O1O2.

2. Giả sử O1O2 cắt BC ở T . Đường tròn (MOT ) cắt OH tại K khác O. Chứng minhrằng tâm của đường tròn (AOK) nằm trên một đường trung bình của tam giácABC .

Bài 3. Xét hàm số f : R→ R thỏa mãn với mọi x , y là số thực thì

f (x2) + f (x y) = f ( f (x)) + y f (x) + x f (y)

1. Chứng minh rằng f là hàm hằng.

2. Khẳng định còn đúng không nếu điều kiện đề bài đổi thành

f (x2) + f (x y) = f ( f (x)) + y f (x) + |x | f (y)?

Bài 4. Cho bảng ô vuông kích thước 100× 100 mà mỗi ô được điền một trong các kýtự A, B, C , D sao cho trên mỗi hàng, mỗi cột của bảng thì số lượng ký tự từng loại đúngbằng 25. Ta gọi hình chữ nhật có các cạnh song song với bảng và bốn đỉnh của nó đượcđiền đủ bốn ký tự A, B, C , D là “bảng tốt”.

1. Hỏi trong các cách điền ở trên, có bao nhiêu cách điền mà mỗi bảng ô vuông1× 4, 4× 1 và 2× 2 đều có chứa đủ các ký tự A, B, C , D?

2. Chứng minh rằng với mọi cách điền thỏa mãn đề bài thì trên bảng ô vuông đã choluôn có một bảng tốt.

50

Page 51: Möclöc - geosiro.com · Sßdöngph÷ìngph¡p˜⁄isŁtrong chøngminhh…nhhåc Nguy„n Tăng Vũ (Giáo viên PTNK TP Hç Chí Minh) Bài toán hình håc phflng thưíng xoay

LÊ PHÚC LỮ

1.2. Đề thi ngày thứ hai

Bài 5. Với a, b ∈ Z, xét hai dãy đa thức P0(x) = x + a, Pn+1(x) = P2n (x) + (−1)nn và

Q0(x) = x + b, Qn+1(x) = −Q2n(x) + (−1)nn với mọi n≥ 0.

1. Tìm điều kiện của a, b để tồn tại n sao cho Pn(x) +Qn(x) chia hết cho x + 1.

2. Hỏi đa thức f (x) = P ′2018(x) ·Q′2018(x) có nhiều nhất bao nhiêu nghiệm?

Bài 6. Xét một tam giác Pascal có 2019 dòng được bố trí theo dạng tam giác đều và vớimỗi số1≤ k ≤ 2019 thì dòng thứ k có đúng k số. Biết rằng ngoại trừ các số ở hàng dướicùng thì mỗi số ở trên sẽ bằng tổng của hai số kề với nó ở hàng liền dưới. Gọi S là số ởđỉnh trên cùng của tam giác và a0, a1, . . . , a2018 là các số ở hàng dưới cùng.

1. Chứng minh rằng S = C02018a0 + C1

2018a1 + · · ·+ C20182018 a2018.

2. Hỏi có bao nhiêu cách gán cho mỗi số a0, a1, . . . , a2018 nhận giá trị 0 hoặc 1 để sốS chia hết cho 1009?

Bài 7. Cho tam giác ABC nhọn không cân có tâm nội tiếp I và (d) là một đường thẳngthay đổi nhưng luôn tiếp xúc với (I) ở K . Đường thẳng qua I , vuông góc với IA, IB, ICcắt (d) ở A1, B1, C1.

1. Chứng minh rằng các đường thẳng AA1, BB1, CC1 đồng quy.

2. Giả sử (d) cắt BC , CA, AB theo thứ tự ở M , N , P và đường thẳng qua M , N , P lầnlượt song song với phân giác trong của góc A, B, C trong tam giác ABC đôi một cắtnhau tạo thành tam giác X Y Z . Chứng minh rằng IK tiếp xúc với (X Y Z).

2. Lời giải chi tiết và bình luận

Bài toán 1. Cho dãy số dương (un) xác định bởi u1 =13 , u2 = 3. Giả sử:

1.1+punun+1p

1+ un

=1+pun+1un+2p

1+ un+1

với n≥ 1. Tìm giới hạn của dãy (un).

2.1+punun+2p

1+ un

=1+pun+1un+2p

1+ un+1

với n≥ 1. Chứng minh rằng lim un = tan2 5π18 .

Lời giải. 1) Ta thấy rằng 1+punun+1p1+un

là hằng số với mọi n nên bằng 1+pu1u2p1+u1

= 2q

1+ 13

=p

3.

Do đó1+punun+1p

1+ un

=p

3⇔p

un+1 =

p

3(1+ un)− 1p

un.

Đặt vn =p

un thì v1 =p

33 , v2 =

p3 và vn+1 =

p3(1+v2

n )−1vn

. Xét hiệu

vn+1 −p

2=

Æ

3(1+ v2n)− vn

p2

vn=

(vn −p

2)2

vn

Æ

3(1+ v2n) + vn

p2 .

Tập san Toán học STAR EDUCATION

Page 52: Möclöc - geosiro.com · Sßdöngph÷ìngph¡p˜⁄isŁtrong chøngminhh…nhhåc Nguy„n Tăng Vũ (Giáo viên PTNK TP Hç Chí Minh) Bài toán hình håc phflng thưíng xoay

52 LÊ PHÚC LỮ

Từ đó suy ra vn ≥p

2 với mọi n ≥ 2 và dễ thấy vn+1 −p

2 ≤ (vn−p

2)2

2 . Bằng nguyên lýkẹp, ta có được lim(vn −

p2) = 0 nên lim un = 2.

2) Ở đây, ta sẽ dùng phép thế lượng giác. Đặt un = tan vn với vn ∈

0; π2

thì

1+ tan vn tan vn+21

cos vn

=1+ tan vn+1 tan vn+2

1cos vn+1

hay cos vn + sin vn tan vn+2 = cos vn+1 + sin vn+1 tan vn+2. Ta đưa về

tan vn+2 =cos vn − cos vn+1

sin vn+1 − sin vn= tan

vn + vn+1

2

hay vn+2 =vn + vn+1

2.

Ta tính được v1 =π6 , v2 =

π3 nên dễ dàng tính được lim vn =

5π18 (do đây là dãy trung

bình cộng) và lim un = tan2 5π18 .

Bài toán 2. Cho tam giác ABC nhọn, không cân nội tiếp đường tròn (O) có trực tâmH và M là trung điểm của AH. Gọi (O1), (O2) là các đường tròn đi qua H và lần lượttiếp xúc với BC ở B, C . Gọi X , Y lần lượt là tâm bàng tiếp góc H của hai tam giácHMO1, HMO2.

1. Chứng minh rằng X Y song song với O1O2.

2. Giả sử O1O2 cắt BC ở T . Đường tròn (MOT ) cắt OH tại K khác O. Chứng minhrằng tâm của đường tròn (AOK) nằm trên một đường trung bình của tam giácABC .

Lời giải. 1) Kẻ đường kính BU , CV của đường tròn (O1), (O2) và đặt R= HU∩AB, S =HV ∩ AC .

Tập san Toán học STAR EDUCATION

Page 53: Möclöc - geosiro.com · Sßdöngph÷ìngph¡p˜⁄isŁtrong chøngminhh…nhhåc Nguy„n Tăng Vũ (Giáo viên PTNK TP Hç Chí Minh) Bài toán hình håc phflng thưíng xoay

LÊ PHÚC LỮ

Ta có BH⊥HU , mà BH⊥AC nên HR ‖ AC . Tương tự thì HS ‖ AB nên tứ giác ARHS làhình bình hành. Suy ra M là trung điểm RS.

Mặt khác, O1 là trung điểm BU và BU ‖ AH nên theo bổ đề hình thang thì O1, R, Mthẳng hàng. Tương tự thì O2, S, M cũng thẳng hàng. Do đó, năm điểm O1, O2, R, S, Mthẳng hàng.

Vì X là tâm bàng tiếp góc H của tam giác HMO1 nên MX là phân giác ngoài của góc∠O1MH, suy ra XR

X H =MRMH . Tương tự thì Y S

Y H =MSMH , mà MR = MS nên XR

X H =Y SY H , điều

này cho thấy X Y ‖ RS hay X Y ‖ O1O2.

2) Ký hiệu (M) là đường tròn đường kính AH. Gọi N là trung điểm của BC thì dễ thấyphương tích từ N đến ba đường tròn (M), (O1), (O2) bằng nhau (đều bằng BC2

4 ). Bađường tròn trên có điểm chung là H và có ba tâm thẳng hàng nên còn điểm chung làL 6= H.

Suy ra L ∈ HN và L ∈ (M) nên theo tính chất quen thuộc thì L chính là giao điểm củatia NH và (O). Do đó OL = OA, mà MA = M L nên OM⊥AL. Ta cũng có AL ‖ O1O2

(cùng vuông góc với H L) nên ∠T MO = 90, suy ra ∠T KO = 90.

Để chứng minh tâm của (AOK) thuộc đường trung bình song song BC của tam giácABC , ta chỉ cần chứng minh (AOK) đi qua chân đường cao D kẻ từ A xuống BC . GọiG là điểm đối xứng với H qua BC thì dễ thấy G ∈ (O). Ta có AON M , MONH là cáchình bình hành nên dễ thấy MONG là hình thang cân. Do đó các điểm O, N , G, M , T, Lcùng thuộc đường tròn đường kính TO. Suy ra

HK ·HO = HM ·HG =12

HA · 2HD = HA ·HD

nên D ∈ (AOK). Ta có đpcm.

Tập san Toán học STAR EDUCATION

Page 54: Möclöc - geosiro.com · Sßdöngph÷ìngph¡p˜⁄isŁtrong chøngminhh…nhhåc Nguy„n Tăng Vũ (Giáo viên PTNK TP Hç Chí Minh) Bài toán hình håc phflng thưíng xoay

54 LÊ PHÚC LỮ

Nhận xét. Bài toán trên còn nhiều cách tiếp cận khác bằng các công cụ như điều hòa,nghịch đảo, biến đổi cạnh – góc, . . . Có thể xem đây là một mô hình “ôn tập” lại nhiềumô hình cũ trong đó. Mấu chốt cho cả hai ý là nhận xét được rằng O1, M , O2 thẳnghàng. Nếu phát hiện các mô hình như: điểm Miquel L, điểm Humpty H, bài toán cóthể được giải nhẹ nhàng và sáng sủa hơn nhiều.

Bài toán 3. Xét hàm số f : R→ R thỏa mãn với mọi x , y thì

f (x2) + f (x y) = f ( f (x)) + y f (x) + x f (y).

1. Chứng minh rằng f là hàm hằng.

2. Khẳng định còn đúng không nếu điều kiện đề bài đổi thành

f (x2) + f (x y) = f ( f (x)) + y f (x) + |x | f (y)?

Lời giải. 1) Thay x = 0, ta có 2 f (0) = f ( f (0))+ y f (0); nếu f (0) 6= 0 thì y = 2− f ( f (0))f (0)

là hằng số, vô lý. Do đó f (0) = 0.

Thay y = 0, ta có f (x2) = f ( f (x)) với mọi x . (*)

Do đó, ta viết lại điều kiện đã cho là f (x y) = y f (x) + x f (y) với mọi x , y ∈ R.

• Thay x = y = 1 thì f (1) = 2 f (1) nên f (1) = 0.

• Thay x = y = −1 thì f (1) = −2 f (−1) nên f (−1) = 0.

• Thay y = −1 thì f (−x) = − f (x) nên f là hàm số lẻ.

Cuối cùng, trong (∗) thay x → −x thì f ( f (x)) = f (x2) = f ( f (−x)) = f (− f (x)) =− f ( f (x)).

Suy ra f ( f (x)) = f (x2) = 0,∀x nên f (x) = 0,∀x > 0; mà f (−x) = − f (x) nên tacũng có f (x) = 0,∀x < 0. Điều này chứng tỏ f (x)≡ 0 là hàm hằng.

2) Câu trả lời là phủ định.

Thật vậy, xét hàm số:

f (x) =

0, ∀x ≥ 0

− x ,∀x < 0

Khi đó, f (x2) = 0 và f ( f (x)) = f (0) = 0,∀x ≥ 0 và f ( f (x)) = f (−x) = 0,∀x < 0nên ta luôn có f ( f (x)) = 0,∀x , điều kiện ban đầu trở thành f (x y) = y f (x) +|x | f (y). Dễ dàng kiểm tra nếu x = 0 hoặc y = 0 thì đẳng thức trên đúng. Ta xét cáctrường hợp sau:

• Nếu x > 0, y > 0 thì dễ thấy tất cả các số hạng đều bằng 0, thỏa mãn.

• Nếu x > 0, y < 0 thì −x y = 0+ x(−y), đúng.

• Nếu x < 0, y < 0 thì 0= y(−x) + (−x)(−y), đúng.

• Nếu x < 0, y > 0 thì −x y = y(−x) + (−x) · 0, cũng đúng.

Do đó, hàm số đã nêu thỏa mãn điều kiện đề bài.

Tập san Toán học STAR EDUCATION

Page 55: Möclöc - geosiro.com · Sßdöngph÷ìngph¡p˜⁄isŁtrong chøngminhh…nhhåc Nguy„n Tăng Vũ (Giáo viên PTNK TP Hç Chí Minh) Bài toán hình håc phflng thưíng xoay

LÊ PHÚC LỮ

Bài toán 4. Cho bảng ô vuông kích thước 100× 100 mà mỗi ô được điền một trongcác ký tự A, B, C , D sao cho trên mỗi hàng, mỗi cột của bảng thì số lượng ký tự từngloại đúng bằng 25. Ta gọi hình chữ nhật có các cạnh song song với bảng và bốn đỉnhcủa nó được điền đủ bốn ký tự A, B, C , D là “bảng tốt”.

1. Hỏi trong các cách điền ở trên, có bao nhiêu cách điền mà mỗi bảng ô vuông1× 4, 4× 1 và 2× 2 đều có chứa đủ các ký tự A, B, C , D?

2. Chứng minh rằng với mọi cách điền thỏa mãn đề bài thì trên bảng ô vuông đãcho luôn có một bảng tốt.

Lời giải. 1) Không mất tính tổng quát, giả sử rằng 4 ô đầu tiên của cột 1 được điềnA, B, C , D. Khi đó, ô thứ hai của cột 2 phải điền D vì nó thuộc hai hình vuông 2×2 đãchứa sẵn A, B, C . Do đó, ta điền tiếp được cột 2 theo thứ tự là C , D, A, B. Cứ như thế,ta điền tiếp cho cột 3,4.

A C A CB D B DC A C AD B D B

Tuy nhiên, ta thấy các hàng khi đó không thỏa mãn vì chỉ chứa hai loại ký tự. Vậy nênkhông có cách điền nào thỏa mãn điều kiện đã nêu.

2) Trước hết, ta sẽ chứng minh rằng tồn tại hai cột sao cho trên đó, có ít nhất 76 cặpô cùng hàng nhưng được điền khác ký tự.

Giả sử phản chứng rằng mỗi cặp cột tùy ý đều có ít nhất 25 cặp ô cùng ký tự. Cố địnhcột 1, xét 99 cột còn lại. Gọi T là số bộ (a, b) trong đó cột a ≥ 2 có ô thứ b từ trênxuống là cùng ký tự.

• Theo giả sử trên thì T ≥ 99 · 25.

• Mặt khác, theo giả thiết thì T = 100 · 24(tính theo hàng).

Suy ra 100 · 24 ≥ 99 · 25, điều vô lý này chứng tỏ giả thiết phản chứng là sai, tức làluôn chọn được hai cột thỏa mãn đề bài.

Tiếp theo, lại giả sử phản chứng rằng không có 2 hàng, 2 cột nào cắt nhau tạo thànhhình chữ nhật thỏa mãn. Xét 2 cột đã chọn được ở trên, giả sử đã có cặp (A, B), (A, C)thì sẽ không có (C , D) và (B, D). Ta có hai khả năng:

• Nếu có (A, D) thì không có (B, C), khi đó mỗi cặp trong 76 cặp đều có ký tự A;trong khi số lần ký tự A xuất hiện trên đó tối đa là 50, vô lý.

• Nếu có (B, C) thì không có (A, D); khi đó, trên 76 cặp sẽ có 76 · 2 = 152 ký tự,trong khi số ký tự tối đa là 25 · 2 · 3= 150, cũng vô lý.

Từ đây ta có đpcm.

Tập san Toán học STAR EDUCATION

Page 56: Möclöc - geosiro.com · Sßdöngph÷ìngph¡p˜⁄isŁtrong chøngminhh…nhhåc Nguy„n Tăng Vũ (Giáo viên PTNK TP Hç Chí Minh) Bài toán hình håc phflng thưíng xoay

56 LÊ PHÚC LỮ

Bài toán 5. Với a, b ∈ Z, xét hai dãy đa thức P0(x) = x+a, Pn+1(x) = P2n (x)+(−1)nn

và Q0(x) = x + b, Qn+1(x) = −Q2n(x) + (−1)nn với mọi n≥ 0.

1. Tìm điều kiện của a, b để tồn tại n sao cho Pn(x) +Qn(x) chia hết cho x + 1.

2. Hỏi đa thức f (x) = P ′2018(x) ·Q′2018(x) có nhiều nhất bao nhiêu nghiệm?

Lời giải. a) Để Pn(x)+Qn(x) chia hết cho x+1, ta cần có Pn(−1)+Qn(−1) = 0. Ta xétcác trường hợp: - Nếu n= 0 thì P0(x)+Q0(x) = 2x+a+ b nên cần có a+ b = 2. - Nếun= 1 thì P1(x)+Q1(x) = (x + a)2−(x + b)2 = (a−b)(2x+a+b) nên ta cần có a+b =2 hoặc a = b. - Nếu n = 2 thì P2(x) +Q2(x) = [P1(x)−Q1(x)] [P1(x) +Q1(x)]− 2.Do đó cần có [P1(−1)−Q1(−1)] [P1(−1) +Q1(−1)] = 2. Dễ thấy hai số

P1(−1)−Q1(−1), P1(−1) +Q1(−1)

cùng tính chẵn lẻ nên không thể có đẳng thức trên. Bằng cách tương tự, ta thấy rằngvới mọi n chẵn và n≥ 2 thì không thỏa mãn điều kiện. Thật vậy,

P2k+2(x) +Q2k+2(x) = [P2k+1(x) +Q2k+1(x)] [P2k+1(x)−Q2k+1(x)]− 2(2k+ 1)

với k ∈ N. Vì 2(2k+ 1) chia 4 dư 2 nên đẳng thức trên không thể xảy ra. - Nếu n = 3thì P3(x) +Q3(x) = [P2(x)−Q2(x)] [P2(x) +Q2(x)] + 4. Do đó cần có

[P2(−1)−Q2(−1)] [P2(−1) +Q2(−1)] = −4

hay (a− 1)4 − (b− 1)4 = −4. Dễ chứng minh được hiệu của hai lũy thừa bậc 4 luônchia hết cho 16 nên đẳng thức trên cũng không thể xảy ra. Xét n ≥ 5 và n lẻ, ta đặtn= 2k+ 1 thì k ≥ 2. Ta có

P2k+1(x) = P22k(x) + 2k =

P22k−1(x)− (2k− 1)

2+ 2k,

Q2k+1(x) = −Q22k(x) + 2k = −

−Q22k−1(x)− (2k− 1)

2+ 2k.

Để P2k+1(−1) +Q2k+1(−1) = 0 thì

[P2k(−1) +Q2k(−1)] [P2k(−1)−Q2k(−1)] = −4k.

Dễ thấy nếuP2k(−1) +Q2k(−1) = 0

thì không thỏa nênP2k(−1) +Q2k(−1) 6= 0.

Chú ý rằng P2k(−1)−Q2k(−1) =

P22k−1(−1)− (2k− 1)

2+

Q22k−1(−1) + (2k− 1)

2≥

(2k− 1)2 > 0, và với mọi k ≥ 2 thì (2k− 1)2 > 4k nên ta thấy

|P2k(−1) +Q2k(−1)| ≥ 1, |P2k(−1)−Q2k(−1)|> 4k

nên|P2k+1(−1) +Q2k+1(−1)|> 4k.

Do đó, trong trường hợp này, ta cũng không thể có đẳng thức. Vậy điều kiện của a, bcần tìm là a+ b = 2 hoặc a = b.

Tập san Toán học STAR EDUCATION

Page 57: Möclöc - geosiro.com · Sßdöngph÷ìngph¡p˜⁄isŁtrong chøngminhh…nhhåc Nguy„n Tăng Vũ (Giáo viên PTNK TP Hç Chí Minh) Bài toán hình håc phflng thưíng xoay

LÊ PHÚC LỮ

2) Ta thấy rằng P ′n+1(x)Q′n+1(x) = −4P ′n(x)Q′n(x) · Pn(x)Qn(x). Quy nạp được

P ′2018(x)·Q′2018(x) = 24036P2017(x)Q2017(x)·P2016(x)Q2016(x) · · · P0(x)Q0(x)·P ′0(x)Q′0(x).

Ta có P4(x) = (((x + a)4 − 1)2+ 2)

2−3≥ 22−3> 0 và Pn+1(x) = P2

n (x)+(−1)n+1(n+

1) ≥ n2 − (n+ 1) > n+ 1. Ngoài ra, Q4(x) = −

(x + b)2 + 12+ 2

2− 3 ≤ −3 và

Qn+1(x) = −Q2n(x) + (−1)n+1(n+ 1) ≤ −n2 + n+ 1 < −(n+ 1). Do đó, ta chứng minh

bằng quy nạp được rằng Pn(x) > 0 với mọi n ≥ 4 và Qn(x) < 0 với mọi n ≥ 4. Ta chỉcần xét n≤ 3. Với n= 0, ta có

P0(x) = 0⇔ x = −aQ0(x) = 0⇔ x = −b

Tương tự với n= 1 vì P1(x) = P20 (x),Q1(x) =Q2

0(x). Do đó, chọn a 6= b thì ta có đúng2 nghiệm thực phân biệt. Với n= 2, ta có

P2(x) = P21 (x)− 1= 0⇔ (x + a)4 = 1⇔ x + a = ±1.

Q2(x) = −Q21(x)− 1< 0

Với n= 3, ta có¨

P3(x) = P22 (x) + 2> 0

Q3(x) = −Q22(x) + 2= 0⇔Q2(x) = −

p2⇔ (x + b)2 =

Æp2− 1

Do đó f (x) có tối đa 6 nghiệm thực phân biệt và cực trị xảy ra khi a 6= b.

Nhận xét. Bài toán gốc yêu cầu tìm số nghiệm của tích

P1(x)P2(x) . . . Pn(x)Q1(x)Q2(x) . . .Qn(x).

Ở dãy đa thức Qn(x), dấu − đã khiến cho bài toán khó chịu hơn rất nhiều. Đặc biệt làđối với ý a, ta cần kiên nhẫn xét các trường hợp để tìm ra quy luật chung khiến chonhân tử x + 1 không thể sinh khi n lớn; ở đây cần kết hợp thêm yếu tố số học cũngnhư đánh giá về độ lớn của các biểu thức. Nhân tử x + 1 có thể thay bởi x + c bất kỳ.

Bài toán 6. Xét một tam giác Pascal có 2019 dòng được bố trí theo dạng tam giác đềuvà với mỗi số1 ≤ k ≤ 2019 thì dòng thứ k có đúng k số. Biết rằng ngoại trừ các số ởhàng dưới cùng thì mỗi số ở trên sẽ bằng tổng của hai số kề với nó ở hàng liền dưới.Gọi S là số ở đỉnh trên cùng của tam giác và a0, a1, . . . , a2018 là các số ở hàng dướicùng của tam giác.

1. Chứng minh rằng S = C02018a0 + C1

2018a1 + · · ·+ C20182018 a2018.

2. Hỏi có bao nhiêu cách gán cho mỗi số a0, a1, . . . , a2018 nhận giá trị 0 hoặc 1 đểsố S chia hết cho 1009?

Lời giải. 1) Ta sẽ chứng minh bằng quy nạp rằng nếu tam giác Pascal có n+ 1 dòngvà các số ở dòng cuối là a0, a1, . . . , an thì số ở dòng trên cùng sẽ là

S = C0n a0 + C1

n a1 + · · ·+ Cnn an.

Tập san Toán học STAR EDUCATION

Page 58: Möclöc - geosiro.com · Sßdöngph÷ìngph¡p˜⁄isŁtrong chøngminhh…nhhåc Nguy„n Tăng Vũ (Giáo viên PTNK TP Hç Chí Minh) Bài toán hình håc phflng thưíng xoay

58 LÊ PHÚC LỮ

Với n= 1, ta thấy tam giác có hai dòng và hiển nhiên S = a0 + a1 = C01 a0 + C1

1 a1.

Giả sử khẳng định đúng với n. Ta thêm vào bên dưới tam giác Pascal có n+ 1 dòngmột dòng mới gồm n+ 2 số là b0, b1, . . . , bn+1. Khi đó, theo giả thiết quy nạp thì

S = C0n a0 + C1

n a1 + · · ·+ Cnn an = C0

n (b0 + b1) + C1n (b1 + b2) + · · ·+ Cn

n (bn + bn+1).

Với 1≤ i ≤ n thì hệ số của bi trong S sẽ là C i−1n + C i

n = C in+1 nên từ đó suy ra

S = C0n+1 b0 + C1

n+1 b1 + · · ·+ Cn+1n+1 bn.

Theo nguyên lý quy nạp thì khẳng định đúng. Thay n= 2018, ta có đpcm.

2) Với p là số nguyên tố lẻ, ta có các tính chất sau:

i) C p2p ≡ 2(modp).

ii) C k2p ≡ 0 (modp) với 1≤ k ≤ 2p− 1 và k 6= p.

i) Ta có:

C p2p − 2=

(2p)!p!p!

− 2=(p+ 1)(p+ 2) . . . (2p− 1)(2p)

p!− 2

= 2(p+ 1)(p+ 2) . . . (2p− 1)− (p− 1)!

(p− 1)!

Tử số đồng dư với 1 · 2 · 3 · (p− 1)− (p− 1)!= 0 theo modp nên C22p ≡ 2(modp).

ii) Do C k2p = C2p−k

2p nên ta có thể giả sử 1≤ k < p. Biến đổi tương tự trên thì

C k2p =

(2p− k+ 1)(2p− k+ 2) . . . (2p− 1)(2p)k!

.

Ta thấy tử số chia hết cho p trong khi (p, k!) = 1 vì 1≤ k < p nên mẫu số không chiahết cho p, tức là biểu thức trên chia hết cho p.

Từ đó ta thấy rằng, để 1009|S thì a0+ a2018+2a1009 phải chia hết cho 1009. Điều nàychỉ xảy ra khi tất cả các số này bằng 0. Các giá trị còn lại ta có thể chọn tùy ý là 0hoặc 1 nên có tất cả 22016 cách.

Nhận xét. Bài toán là một mở rộng thú vị của tam giác Pascal thông thường, giá trịcủa tổng S bằng tổng của các số nhỏ với hệ số tương ứng bằng các hệ số nhị thức. Kếtquả bài toán vẫn đúng khi thay 2018 bởi 2p với p là số nguyên tố lẻ bất kỳ.

Bài toán 7. Cho tam giác ABC nhọn không cân có tâm nội tiếp I và (d) là một đườngthẳng thay đổi nhưng luôn tiếp xúc với (I) ở K . Đường thẳng qua I , vuông góc vớiIA, IB, IC cắt (d) ở A1, B1, C1.

1. Chứng minh rằng các đường thẳng AA1, BB1, CC1 đồng quy.

2. Giả sử (d) cắt BC , CA, AB theo thứ tự ở M , N , P và đường thẳng qua M , N , P lầnlượt song song với phân giác trong của góc A, B, C trong tam giác ABC đôi mộtcắt nhau tạo thành tam giác X Y Z . Chứng minh rằng IK tiếp xúc với (X Y Z).

Tập san Toán học STAR EDUCATION

Page 59: Möclöc - geosiro.com · Sßdöngph÷ìngph¡p˜⁄isŁtrong chøngminhh…nhhåc Nguy„n Tăng Vũ (Giáo viên PTNK TP Hç Chí Minh) Bài toán hình håc phflng thưíng xoay

LÊ PHÚC LỮ

Lời giải. 1) Giả sử (d) tiếp xúc với (I) ở K . Gọi D, E, F là tiếp điểm của (I) với cạnhBC , CA, AB. Gọi V là hình chiếu của K lên A1I và U = AI∩EF . Khi đó dễ thấy IU · IA=IV · IA1 = r2 (với r là bán kính của (I)).

Xét phép nghịch đảo Ω tâm I , phương tích r2 thì A↔ U , A1↔ V nên AA1↔ (IUV ).Xây dựng các điểm B′, C ′ tương tự thì ta đưa bài toán về chứng minh các đường trònđường kính IA′, IB′, IC ′ có điểm chung khác I .

Tuy nhiên, A′, B′, C ′ cùng nằm trên đường thẳng Simson của K đối với tam giác DEFnên xét phép vị tự tâm I , tỷ số 1

2 thì tâm của các đường tròn trên thẳng hàng. Do đó,chúng có điểm chung khác I ; ảnh của điểm đó qua phép nghịch đảo chính là điểmđồng quy của ba đường thẳng đã nêu.

2) Xét vị trí tương đối của các điểm như hình bên dưới, các trường hợp còn lại chứngminh tương tự. Gọi S là điểm đối xứng với I qua K . Ta có Y Z ‖ AI , Y X ‖ IC nên∠MY P = ∠X Y Z = (Y X , Y Z) = (IA, IC) = 90 − ∠B

2 . Mặt khác, ta có

∠MSP = ∠M I P = ∠KI M −∠KI P =∠KI D−∠KI E

2=

12∠DI F = 90 −

∠B2

,

suy ra M , P, Y, S cùng thuộc một đường tròn hay S ∈ (M PY ).

Chứng minh tương tự thì S ∈ (N PX ), S ∈ (MN Z). Từ đây suy ra S là điểm Miquel củatứ giác toàn phần gồm 6 đỉnh M , N , P, X , Y, Z . Do đó S ∈ (X Y Z).

Tiếp theo, để ý rằng bằng biến đổi góc tương tự trên, ta có hai tam giác X Y Z , DEF làđồng dạng thuận. Do đó, tồn tại phép vị tự quay Ω biến X Y Z → DEF với góc quay90 (do các cạnh của hai tam giác tương ứng vuông góc).

Tập san Toán học STAR EDUCATION

Page 60: Möclöc - geosiro.com · Sßdöngph÷ìngph¡p˜⁄isŁtrong chøngminhh…nhhåc Nguy„n Tăng Vũ (Giáo viên PTNK TP Hç Chí Minh) Bài toán hình håc phflng thưíng xoay

60 LÊ PHÚC LỮ

Kẻ đường kính KG của (I), ta có ∠GDE = ∠GKE = ∠IKE = ∠KN I = ∠KNS =∠PXS = ∠SX Y ; tương tự thì ∠GED = ∠SY X . Suy ra Ω : S → G (hai điểm tươngứng). Gọi J là tâm X Y Z thì Ω : J → I nên ta phải có JS⊥IG. Mà các điểm I , G, K , Sthẳng hàng nên ∠JSK = 90 chứng tỏ IK tiếp xúc (X Y Z).

Nhận xét. Ngoài cách tiếp cận bằng phép nghịch đảo cho câu a, ta còn có thể dùngcực – đối cực một cách tự nhiên hơn. Ở câu b, mấu chốt vấn đề là phát hiện đượcđiểm Miquel là tiếp điểm của IK với đường tròn (X Y Z). Thực ra có thể bằng các biếnđổi góc phù hợp để chỉ ra

∠XSK = ∠X ZS

trực tiếp nhưng hơi rắc rối, thiếu tự nhiên.

Tập san Toán học STAR EDUCATION